Passage of the game deadly space. Walkthrough Dead Space. Basic character skills

Introduction:(a few rules for a pleasant passage)

  • When you meet a monster, do not waste ammo, it will be most effective to shoot them off limbs, i.e. arms, legs, head. How do you know if you have killed a monster? Shoot off all limbs and it will be 100% corpse =)
  • Do not forget to collect resources and items, they will be needed to pump ammunition and weapons.
  • Some huge monsters have weak spots, they are usually highlighted and colored yellow-orange (ocher). Just aim gently there and shoot him all ^_^
  • Oh, I almost forgot, you will probably be wondering where to save us. So, the saves in this game look like boxes that resemble a camera hanging on the wall.
  • On my own behalf, I will say that the first best weapon is plasma cutter, but nevertheless, for killing a large amount of meat running at you, it is suitable plasma rifle.

The game has 12 chapters:

  • Chapter One: Arrival
  • Chapter Two: Intensive Care
  • Chapter Three: Orbital Correction
  • Chapter Four: Death Is Inevitable
  • Chapter Five: A Deadly Addiction
  • Chapter Six: Dangerous Impurities
  • Chapter Seven: Step Into the Void
  • Chapter Eight: Find and Rescue
  • Chapter Nine: Dead on Arrival
  • Chapter ten: Last days
  • Chapter Eleven: Alternative Solutions
  • Chapter twelve: Dead space

A little preface.

Year 2507. The resources on Earth are completely depleted. There is only one way for humanity to survive - space, search for resources on other planets. In the course of space exploration, the planets found become an ideal source of minerals. Huge cargo ships - rippers are draining planet after planet. One of the ships - Ishimura did not get in touch ...

A group of specialists on the Callion shuttle is immediately sent to find out the reason for the disappearance of the connection.

Well, let's start the game!

Chapter 1: Arrival

CEC Mission ER529

Status….

USG KELLION, COURSE - AEGYD SYSTEM ...

The main crew consists of 5 people

  • Isaac Clarke - main character - engineer (technician), specialist in ship systems
  • Kendra Daniels - Computer Systems Specialist
  • Zach Hammond - Senior Security Officer
  • Corporals Chen and Johnston - pilots

DIRECTIVE A: LOCATE PRODUCING SHIP USG ISHIMURA ...

DIRECTIVE B: INSTALL AND REMEDY CAUSE COMMUNICATION PROBLEM ...

Our protagonist (s) Isaac sits quietly, listening to the discussions of the crew on the Callion shuttle. It did not work to get in touch with Ishimura. After landing, just follow your team and fulfill their requests. You will be asked to open the door, to do this, go to the door and press “ E" (default). If you are lost, click " V”, She will show you the way. Next, you will need to activate the console. Expect a cute scene) Your task is to run! And as quickly as possible, there is nothing to fight with. Your salvation is the elevator. So you met the monsters - necromorphs.

Necromorph - (Greek morphe - form, necro - death) - an alien life form, most often found in the form of mutated corpses. Roughly speaking, the necromophilic bacterium enters the dead organism and instantly begins to multiply, leading to the mutation of the entire organism. As a result, the corpses come to life and change the shape of the body - fangs, blades, tail, etc. appear. Organs are also mutated. If you want to kill a necromorph, shoot them limbs.

This is how the necromorph looks like =)

When you go downstairs on the elevator, you will find yourself in a room with a corpse, not far from it there is an eloquent inscription "Cut their limbs"... As I said above, it is better to do just that. On the table you will find your first weapon (Congratulations!) - the Flame Cutter. Perhaps the best weapon for instant amputation of limbs. Trample corpses and boxes, you will find a lot of useful things in them.

The flame cutter has 10-18 damage, its ammunition capacity is 10-20 rounds, the reload time is 1-0.5 sec. rate of fire 1-0.6 sec. per shot. It also has an alternate mode (vertical). This is a 90 degree turn. Suitable for upgrading on a workbench

After passing in front, you will see Kendra and Zak, they are the only ones who could survive. You will be asked to fix the transportation system. Don't forget to save on the way. For the future, not all corpses are corpses ) therefore, if you notice a strange body on the floor, it is better to shoot at it from afar, this will greatly facilitate your life. Otherwise, you will, like me, jump on the chair, hearing the beautiful voice of the necromorph behind your back ^^ Don't miss the resources that are hidden in the boxes. Because in the future you can improve your things on the workbench. The first obstacle is a jammed door, we raise the stasis module, a very useful thing, we aim at the door and press " WITH", And we can pass, just do not delay, because stasis slows objects down for a while.

The stasis module has a charge level that needs to be replenished. With the help of stasis, you can slow down not only doors, but also objects and opponents that bother you a lot, use it when you do not have time to kill monsters running at you. Stasis will also help to move some parts in the game. Suitable for upgrading on a workbench

You find yourself in a room where you need to start the mechanism. We kill everyone. Move forward.

In one of the rooms you will find a power unit on the wall, ammunition in the boxes, and an information panel on the table. There is also a workbench in the room.

Workbench - a set of devices for improving the parameters of the GG. With its help, you can improve all the weapons that you have with you, your suit - X or stasis module. Power nodes are needed to improve. And also you can lay out unnecessary things. (it works like a warehouse.)

We come back. A good portion of monsters on the way is provided to you = P Then we often use "B" and go forward to adventure. On the way, we deal with the necromorphs. After breaking up the Necromorph Demonstration, head to the boarding terminal and then to the boarding station, exploring the restroom and smashing crates along the way. We go to the freight elevator, which will quickly take you down. There will be a shop on your right.

The store is the only one in Dead space a way to buy, not find, ammunition, first aid kits, weapons, power nodes. In addition, in the store you can buy new costumes, which will become available as you progress through the game.

Having bought all the money with shaking hands, we go to the trolley. She will bring you to the next chapter.

Chapter 2: Intensive Care

At the exit from the trolley we see a woman, do not be afraid, come, she will give you a useful device - a kinesis module.

Kinetic Pistol - Allows you to move objects at a distance, including picking up objects and throwing them at monsters. Suitable for upgrading on a workbench.

For the passage further use the received device. Come forward. You will be taken to a room with three doors. One of them, leading to the intensive care unit, is barricaded and will have to be blown up. The components of the explosives - glycerin and an electric pillow - are to be found in the remaining two doors. Search all rooms and move forward. Do not be very scared when the light turns off and the siren starts beeping, just take a comfortable position, you will have to fight with more than one monster, I advise you to aim better. We are moving forward with battle (do not forget to use the " V"). You will be able to breathe when you hear that the quarantine has been lifted. We explore all the premises. We get into the office, take the video there, search everything, the cabinets can be pushed aside with kinesis, so you will get access to secret room where you will find a gold semiconductor - a very high value that can be sold in a store.

On the ground floor you will meet the new kind a monster that you are destined to kill. Further on the elevator we rise higher. This will be the main laboratory, here you can find explosives and a power unit. Having searched everything, we return back. Remember to reload your pistol = P On the way, we save and shoot back, the game invites you to the diagnostic wing.

(scene) - I'm going to kill myself against the wall ... This phrase is great for our corpse.

We move forward to the elevator. It is de-energized, but this is not a problem, we use a nearby power source. It must be inserted into the socket on the wall with a lightning bolt. Before entering the elevator, inspect all rooms. Then we boldly stomp into the elevator and go up to the second level. Climbing up, you will find yourself in a large room with a mechanism. Your task is to move this mechanism using kinesis. Prepare to shoot back. After disassembly, you will again have to move the mechanism (platform), now take the elevator and go further. Fun! Fun! Fun! = P

Here you will have a walk in space, but do not be afraid - the air supply in your spacesuit will not let you die, but oxygen may run out, so you should not count the crows. There are usually oxygen stations in such places. Another door, and now you are in the usual corridors. Further - an interesting and huge hall with a door at the opposite end and a huge abyss in the middle. The only way get over - turn off gravity, fortunately, the remote control is nearby. Now, when you jump, you can gain a foothold on any surface - the magnetic boots of your suit will provide a secure grip. Zach will tell you that. We feed the door, as we did before, move forward, collecting everything along the way forward. We picked up an electric pillow. We are great, but after leaving the room, we need to defend ourselves again. We repeat the manipulations with the block. By the way, here you will get acquainted with a new type of monsters - larvae (Swarmer). It's easier to kill them with fire, if they jumped on you, click " E", But they bite painfully. Moving forward into the large room, here you can change your costume or get blunt. Blow up the barrier. Zach will tell Isaac to go to the morgue. We go to the morgue, killing all inanimate. You probably wanted to know how these monsters are made? One of the ways - a monster that resembles a bat, digs its trunk into the brain and infects the body with bacteria, which transform the corpse, reviving it. All this you will see when you get into a large room with a glazed office. It is advisable to kill the resulting type of monster as soon as possible, it is quite nimble and strong. If you can't keep up, don't spare the stasis. And another tip - kill this mouse first, otherwise it will make a lot of necromorphs. We come back. We are in the engine room, the path to which again runs through the Transport Deck.

Chapter 3: Orbital Correction

Arriving at the next place of events, you will again receive instructions:

  • It is necessary to restore the gravity on the ship;
  • It is necessary to restore the fuel supply to the engines.

You will have to kill several necromorphs, after which in a small room on the first floor you can find ammunition with a diagram, there is also a workbench. Craft. In one place (use the locator - " V») You can use kinesis to pull the switch down, thus activating the refueling mode. After wandering through the dark corridors and already getting used to the stomp somewhere near a wandering monster, you can find a power node. Moving forward, saving. Finally, we find the second lever. You are informed that fuel is being supplied. Now you need to go back, but the path will not be easy. A large number of mutants await you. Your further path leads to the centrifuge, where you can get by elevator through another door of the control room. Before you enter the room, thoroughly search the closets and pick up the ammunition. In the room, by pressing a button, you start the disinfection process, this will attract the attention of a huge number of mutants that will constantly appear in the room, using ventilation pipes to move. To cope, use stasis and severed limbs of enemies. Ahead of you is a zone of zero gravity with monsters who are not averse to lowering your health. We deal with them. To restore the energy supply, it is necessary to freeze the large rotating shafts with stasis and immediately use kinesis to insert them into the center of the centrifuge. Power up the centrifuge from the console above: gravity will return and all the debris will be sent into outer space. Moving forward. And here we are building a house of bricks. Who cares for what. Get ready for a giant tentacle attack that will grab you. To get rid of the grab, shoot the yellow-orange growth on it. We run back along the corridors, killing the necromorphs. We need to go to the engines. We are moving in the direction of the locator. He will lead us to a large room, where we activate the engines. A massive attack of monsters awaits you, be on your guard. Go back to the computer to confirm the ignition. Now that the engines are running and the orbit has been adjusted, a new problem awaits you - the ship is moving in a field of asteroids that can destroy the ship, and the defense is deactivated. Isaac needs to go to the cart. Hammond will be waiting for him on the bridge.

Chapter 4: Death Is Inevitable

When you meet with Haymond, in his words, it is in his power to repair the control of the anti-meteorite defense weapon, but this requires energy, for which three key nodes are responsible, which Isaac will have to repair. But first you need to activate the elevators through the security room. You can certainly continue playing Dead Space and go through the open door, but I advise you first of all to search the bridge for useful items. You need to return to the large hall where the meteorite fell. Here you will meet a new, very dangerous type of monsters - Brutus.

Brutus- This is a huge necromorph, covered with a thick layer of armor (reminiscent of the shell of a crab), which is impenetrable to conventional weapons. The only way to defeat him is to shoot at the shoulders, or rather, at the yellow growths on them. The ideal option is to freeze with stasis and slowly, tastefully cut the carcass. By the way, don't forget to search it later.

Use stasis more often, you have a slight advantage - the monster moves slower than you. You also have a hint from me personally - if you manage to get around him, then there are already familiar yellow-orange growths on his back, aiming at them, you will kill him much faster. Search all rooms. Move further along the corridor, destroying everything along the way and carefully bending around the bluish rays beating upward - they are deadly. Deal with Brutus again. Ahead there are two electrical cables waiting for you, under which I do not advise you to fall. To get past them, you need to use stasis. There you will find the first node.

To get to it and turn it on, you need to freeze the cables. Return to the elevator from which you came, and return upstairs (if you forgot the way, then use the locator). This time in the hall you have to deal with a considerable number of opponents. Take another lift in the center. At the exit of the elevator, you will find an audio log and a diagram. On the walls you will find ammunition and a save point, and a little further - a couple of strong opponents. Behind them is node 2.

After that, we go into the elevator. When you find yourself in the large hall, prepare to shoot back. Examine the hall carefully, and also go into all the doors. In addition, there is a door closed with a power lock (you will have to spend a power unit), behind it there is a lot of ammunition. Continue straight through the hall, at the end you will find the third node.

Well, the anti-meteorite weapon is ready for firing, it is only necessary to repair the data cable in order to aim fire at asteroids. We use the locator, we will have to go back, having climbed several elevators and killed the monsters in the hall. You need to get out onto the hull of the ship outside. Zack gives good advice - look for shelter so as not to be killed by asteroids. Remember that the air supply is limited. You can also move by jumping between objects at a great distance. Going into the airlock, take a semiconductor on the left, you can also save. Your finest hour !!! Sit in a chair, we will check your accuracy and reaction =) After a few misses, and maybe even mates xDD the auto-aim will turn on, and you can go back to continue the passage.

This time there will be no meteorites on your way, but there will be several necromorphs who will try to kill you. It would seem that you can finally breathe easy, but it was not there. Soon you won't be able to breathe at all. Something poisons the air and your next task is to find the necessary medications to stop the creature. Hammond goes to the hydroponics department to fight the creature, and in the meantime you need to find the means to fight. You need to repeat all the way back and return to the transport station. After killing a couple of monsters and reaching the trolley, you will complete the passage of this stage.

You can take a break = P

Chapter 5: Deadly Addiction

The passage of the chapter begins with a communication session with Kendra. The contaminated air is spreading faster than she bargained, she said. The only hope to survive is to prepare chemical weapons for the air poisoning creature. And you need to do it quickly. You can find the necessary ingredients in the chemical laboratory. You passed next to her in the second chapter, but this time Kendra will break open the door leading to the laboratory for you. Thank you very much.

Follow the trodden paths to the room with three doors. There you go through the second door on the right "Imaging Diagnostics Wing", judging by the voices that are heard in the room, you are not the only survivors on the ship. Further along a long corridor. It will lead you to the diagnostics room, where you will meet a new type of monster. , you will need to use the stasis. Next we go to the chemical laboratory. The creatures in the tanks clearly hint that someone experimented with the source material of the necromorphs, combining it with the human, and against the will of the latter. In the center of the room there is a device for creating chemical mixtures, use it, to get a chemical capsule ( E to take). The glass will suddenly open and you will see a mad doctor who decided that the days of humanity are long over and it is time to move on. The result of this "movement" will suddenly come to life in one of the cameras. It is useless to fight him - you simply cannot kill him, the severed limbs will be restored. The only way is to run after freezing the creature with stasis and cutting off its legs. Quickly run out of the room, the creature will remain there for the time being, and you will again face small necromorphs. Go to the clinic, so as not to get lost, use the map or locator, you will meet with the doctor again. Leave the room, you will find yourself in a large hall, where HELL is waiting for you ( xDD): in addition to the regenerating necromorph, you have to destroy a huge number of creatures. Use stasis actively and try to delay the immortal necroforph until Kendra opens the door lock. (Here is an asshole, you think) And again a dark corridor. At the end of it there will be another creature on the wall, to the left of it, in the corner - a semiconductor. Finally, you will reach a small room filled with jars of tissue samples, next to an audio magazine, ammunition and semiconductor on the floor. You will find a device in the room, load a chemical capsule into it, and in a few seconds you will receive an almost ready-made chemical, but it must be taken to the chemical laboratory again. In the meantime, you need to continue the passage of the game Dead Space and get out of the compartment quickly - the abnormal doctor "pumped out the air", and you need to go back, making your way through large groups of necromorphs. To avoid getting lost, use the map and locator. It's a long way, and there is little air, so move on. After reaching the room with three doors, get to the system responsible for the air supply, turn it on. Now your way again lies in the chemical laboratory. To be happy, I will inform you - the way there will be free, mix the ingredients in the device and get the final chemical weapon. Now you need to return to the transport station. The mad doctor will contact you. He will tell you about his plan. A new door will open - for you there, on the wall - a save point, in a large row of cabinets - ammunition. In the large room you will learn that this psychopath wants to transfer the infection to Earth, but until you can stop him, it's time to deal with his immortal necromorph. It is simply impossible to kill him, but to freeze him forever is completely. Your task is to lure him to the freezing station. The task is to run at breakneck speed and freeze the monster with stasis. Run to the room upstairs, there is a cryo-freezing control center, press the button on the console and watch how your almost invincible enemy goes into a long freeze, there he is a road. Search the room. You can breathe out, the second door leads you almost to the transport station.

Chapter 6: Dangerous Impurities

At the station, pick up the audio recording, it will give you an initial information about what is happening in the hydroponics department. Moving forward, taking the elevator, you will meet barely alive Hamond. He will tell you that he was able to find out that there is some huge creature in the food storage. This is the root cause of air pollution. Hydroponics deck workers have mutated into air pollutants. If you want to survive, you will have to kill all of them, more precisely - eight, and only then destroy a huge creature - Leviathan.

Leviathan is a huge creature. The body is round, there is a semblance of a mouth, three tentacles. Yellow mines fly out of the mouth, which explode on impact, they can be shot down from a distance. (The Pulse Rifle and Plasma Cutter have proven to be excellent weapons against the Leviathan.)

You will be taken to a room with several doors. One of them, leading to the food storage, is still closed, the other leads to the hydroponics department, where the infected workers are located.

Follow the prompts on the map or locator and move on. Try not to get caught in the fumes of the gas. We pass forward, take the elevator, and we find ourselves in the "Western greenhouse". From now on, be extremely careful and careful, use the locator.

You find yourself in a huge room that is fraught with many dangers, monsters and supplies. The locator will lead you to a room with an infectious agent in the middle, a sure sign is greenish air and Isaac's use of an oxygen tank. It is not necessary to shoot at the creature, it is enough to stomp on it.

Again a large hall and a similar room at the other end of it, the ending with the contaminant is the same.

Take the elevator, which is in the middle of the hall, go up to the third, topmost floor, you can find a semiconductor and some ammunition. It is better to do this before visiting the second floor, since then the road will not return here. Go down to the second level. The locator will lead you back to the infected room, at the end of which the third infestator will sit on the left. However, once you kill him, a surprise awaits you. A couple of creatures will appear that can revive corpses and turn them into necromorphs.

Kill them quickly before they create an army against you. Your task is to get to the room on another

side of the hall. Be careful, here you have to face the Fuse.

If you shoot at the yellow growth on your hand, it will explode, so don't do this when the enemy is at close range. You can cut off your hand with a bomb and throw it at other opponents. Hell goes on. You will be taken to a large room with no gravity. To continue moving, you need to remove the flap from the ceiling using kinesis. There will be many necromorphs behind it. Having finished with them, go straight to the hole you have opened. We use the locator, move forward, do not forget to interact with the switches.

When you get into the room with the pipes, be extremely careful, I hope you shouldn't say that you don't need to go into the pipe when there is a fire in it. Tip: you can turn off the switches (of the second and third pipes) while in the doorway. Use the locator and move forward. As soon as you go through all the pipes blazing with fire, entering a small room, you will find the next infestator.

We move along the locator to the second greenhouse. We make our way forward along the corridor, there mutants are waiting for us. If possible, we use the power unit and open the door, take away the sweets. We run to the elevator, we go to the higher level. You will face Brutus, and a little further there will be another infestation.

The next contaminant hid from us among the oxygen tanks.

It is immediately necessary to activate the cylinder replacement system. Take the elevator to the third level. The Penultimate Infector awaits us there. We use the locator and move to the last infection.

screenshots of infestations: (from left to right)





When you meet the tentacle, remember my advice =) shoot him in the growth. We run along the corridor, the locator leads us to the zone of zero gravity - Caution! A drum with electrical energy blocked our way, to get through, just use stasis.

Climb up, there in the corner you will find the last infection. To proceed further, you need to restore the oxygen supply. We come back. The way back will be practically free. The locator will guide you to the console, confirm the activation of the air recirculation system. The door leading to the food storage will become available. Insert the flask with chemicals into the device, the door in front of you will open.

Say hello to the friendly Leviathan.

Do not be afraid, come closer, just collect all the supplies first ^^

Liviathan will want to stroke you with its tentacles (behind the scenes - what a lovely tentacle :R)

This is one of his attacks, the second - he will spit mines at you. It will not be difficult to cope with it if your vigilance and attention does not let you down. We shoot at mines and growths. You can play pingpong with Liviathan's mines: we catch them and throw them at him. Fun!! Try to move more often as the blows of the tentacles are quite painful.

After the victory, return to the transport deck. Kendra has an escape plan. An SOS signal must be sent. Let him be free and clean, calmly walk around the ship.

Chapter 7: Step Into the Void

This chapter begins with a connection with Kendra. The situation is sad, you need to find a radio beacon capable of transmitting a signal to the rescue ship, attach it to the asteroid = P

This asteroid was specially prepared for melting. You have to send it into outer space to receive a high-quality and strong signal.

The control room is closed, but don't worry, somewhere here, wandering around this chapter in our favorite game, you will find a spare key. (Just like in Pinocchio, everything comes with difficulty) Well, let's go look for this key, I hope you are ready ) Go…

Take you straight down the corridor until you reach the door. There will be a power node on the left. Behind the door there will be, no, no, do not be afraid, this is just an impressive corridor in which you will find a lot of useful things for yourself, look carefully. There will be an elevator at the end of this corridor. The lift can take you to four decks: 1 ( A),2(B),3(C),4(D). You yourself are on the first - A, the game takes you to the second deck - V... I advise you not to lower the gun, you have to fight. Because monsters will jump on Isaac from the roof. Going down to the deck, go straight and left - waiting for you another series attacks of monsters. Use " V"For a reference point. Near the save point, turn left and go down. A break in the wall will lead you to a zero gravity zone. According to Kendra, you need to restore gravity. But for this it is necessary to throw the meteorites that are in the room (4 pieces, the quantity sensor is on the wall) straight into the flame that blazes in the room.

To do this, drag the meteorites using kinesis. Get ready to repulse the attack of the necromorphs, there will be a lot of them in the hall. When you destroy the meteorites, gravity will appear, and with it new opponents. Next to the gravity activation panel, you will find a room that requires a power node. Once inside, you will find some ammunition and a semiconductor. In the control room on the opposite side you will find an access key - it will be useful to you further in the passage of Dead Space, this is the same key.

Go to the second door - it will be faster this way, there will be a power unit on the right side of the wall. Soon you will reach an elevator leading to other decks. Use it to go down to the deck - D... Be careful - there are a couple of necromorphs waiting for you that are able to revive corpses. It is advisable to deal with them quickly. Search the room carefully (you will find a gold semiconductor in the corner). Go into the door, use kinesis to pull up the trolley. As soon as you press the start button, immediately carefully look around - there will be necromorphs that can shoot at you. Since their number is large, and there is nowhere to run, you need to deal with them quickly.

After reaching the end, Isaac will start having glitches - he will see Nicole. She will help open the room. Protect Nicole.

Use kinesis to pull supplies towards you. Take all the supplies in the room and the lighthouse. In addition, on the rack in the room there is a diagram of the fourth level protective suit - it can be of great use to you.

We come back. We shoot back and don't get scared =) We take the elevator to the deck - WITH.

Your task is to disable the gravitational clamps holding the asteroid.

We use the locator, move forward, fighting off the necromorphs. You can get to the elevator along the corridor, but first take a look to the left - here you will find a workbench and a shop. You need to take the battery and carry it to the opposite end of the level. There will be a de-energized socket where the battery must be inserted. To find out the location, use the map. After inserting the battery into the slot, go downstairs on the elevator next to it. Be careful, Demoman will attack you. A long corridor will lead you to a considerable supply of ammunition and a gold semiconductor.

When you find yourself in the largest hall with zero gravity, you will need to turn off four disks. Two of them are in the hall, two are outside. You can get out by jumping on an asteroid, and after reaching a vacuum on it, do not forget to install a radio transmitter on the top of the asteroid (in a specially designated place). To stop the disc, you need to stand on the side and shoot at the inside, it is covered with a metal rotating plate, it can be frozen. All this time you will be bothered by the necromorphs. Then we use a map or a locator, go back, just carefully. We reach the elevator and go up higher. We reach the door of the office of the management of the extraction of minerals, we use the found key. Don't forget to collect all the supplies. We activate the console. Now the signal is being broadcast. In addition, there is another problem - the transmitter works, but there are problems with the receiver. You need to go to the bridge and, if possible, fix it. But the seemingly already safe path back to the transport module suddenly ends - quarantine is turned on in the office, and monsters begin to crawl out of the ventilation holes from everywhere. Knead them all into porridge)

Having fun, we go to the deck - A. Be on the alert, it will be unsafe in the elevator... Using the locator, we return to our favorite transport deck.

Chapter 8: Find and Rescue

Uraaa! You have been heard, a warship is flying to save you. The only problem is they can't hear you. Kendra asks you to fix this. You need to get to the bridge and from there to the communication center. Don't forget to shop in stores and use your workbench. There will be many necromorphs on your way, especially black ones, they are stronger. We deal with them and go to the elevator, to us on the third level. Having passed a little further along the corridor, you will find a new type of opponents - Divider (divided), it is easy to recognize him by his high growth and thinness, after death this creature will disintegrate into many small ones, each of which strives to kill you, it is good to fight them with weapons of mass destruction.

Moving forward using " V", We reach the trolley, launch it. We go forward on the assignment. Through large circular doors you will reach the zero gravity zone. Pay attention to the wall - see a schematic of working (green) and non-working (red) transmitters? You need to use kinesis to rearrange the transmitters so that there are only working transmitters in the first circle. That is, the technology is as follows - from the first circle, with kinesis, we throw out the non-working (broken, dented) transmitters and instead of them we install the working ones with the same kinesis (with blue illumination). Elementary =)

We return to the panel, send our coordinates. After activating the panel, you will learn that the ship that was supposed to save you has taken on board a lifeboat with a necromorph. I don’t think it’s worth explaining what could be happening on board the rescue ship right now? Perhaps you have a chance to warn them. But the signal is very weak - something organic and definitely huge has covered the entire communication area outside the ship. Ride the cart back. Instead of an open elevator, you will see a black necromorph, having dealt with him, calmly go downstairs. Take the freight elevator up. Now sit down behind the turret.

How do you like the carcass? ))

Shoot the monster once to make it react. Tentacles with yellow-orange growths will appear, and you need to shoot at them, just beware of the debris and barrels flying at you, they will spoil your ship. On the right is an indicator that shows how many percent of the protective shield is left, do not let it fall to zero if you do not want to start the passage from the next save point. When you kill the creature, Kendra will try to contact the rescue ship, we can see from the video on the screen that it is already too late. The ship crashes into Ishimura and Isaac loses consciousness. Finally, Hamond gets in touch, someone, according to him, blocked the signal. He found a work shuttle, which can be used to escape from Ishimura. To check this, you will have to go to the cargo compartment. Go down the elevator, then along the corridor to the second elevator, down again, you will find yourself on the deck, here you will find another dividing enemy. Repeat the path back to the transport bay.

Chapter 9: Dead on Arrival

The chapter begins with a dialogue with Hamond, he will delight you with one piece of news. The same rescue ship - Waylor, it was not by chance that I ended up here. The ship was equipped for a combat operation. Unfortunately, it is not yet possible to find out everything to the end ... again, interference. After Kendra will contact us, she will tell us that she lost him again (she meant the connection with Hamond). We need to find the converter and get out of here. Let's go ahead. You enter the zero gravity zone, you are required to throw the flying green koloboks into open space.

But first of all, we go down to the bottom and destroy the locks of the hatch. Prepare to fire back and replenish oxygen as needed. Don't forget to collect resources. When you have dealt with radioactive materials, the locks will be unlocked, so as not to get lost, use the locator. Kendra will contact Isaac again. She will tell you what you need to find. The task is to find the necessary parts in the engine compartment, at the stern. Naturally, you will not get straight, you will have to go around. Well, let's go. Once in the room, we are afraid of hustlers (advice: when attacking with them, use stasis if problems arise) I think that you will master the path, your task is to move the boxes with kinesis, I advise you to check every corner. Oh yeah, if you overdo it while killing these hustlers, I suggest relieving stress on the corpses. =) maybe what will fall out xDD Okay, when you shake your heart out, go ahead. Save, the lights will go out, and two necromorphs will appear in the room, capable of reviving corpses. Do not let them do their dirty work. After that, you need to power the elevator with a battery. While you are in the elevator, you will be contacted by a survivor - Dr. Terrence Kane. Indeed, Waylor's crew knew much more than you did when they arrived on the damned Ishimura. The marker is the source of everything that happens, until it returns to the planet where it was taken from, hordes of necromorphs will threaten all of humanity. The shuttle, which you may be able to repair, must be used to return the marker to the planet, only then this incessant nightmare will be over. Go around the blockage, turning to the right, a little to the right there is a small room with supplies, at the end of the corridor there is a door. Behind it you will find a splitting monster, explosives and a power node on the left wall. There will be a shooting gallery nearby.

You can have some fun here.

We strongly recommend that you practice your skills in it. And here's why: there are 5 levels in the shooting range, for each level you pass you will receive a prize. The first level is cartridges, the fourth is a semiconductor, and the fifth is a power unit. To win, you need to shoot at the red pieces, don't touch the blue ones. Do everything quickly and clearly and significantly improve your position. Having played enough, it's time to return to the passage =)

We move forward and find the very martyr ... Immediately after him we enter the room. I DO NOT ADVISE hit by lasers. Using the locator, move forward, keep the cannon ready. There will be a lot of kills in the next 3-5 minutes ^^

After passing a long corridor with an impressive number of necromorphs, you will soon reach the engineering compartment. Collect supplies. In the middle of the corridor there is a turn to the right, take the open elevator upstairs. You will be taken to the boogag hall of hellfire ^^ So, your task will be kinesis to push a huge engine O_O or something similar to it, it will protect you from fire, provided that you stand as close to it as possible. On the side there will be switches that need to be shot, otherwise you will fry Isaac (advice: if you still fry the poor fellow, Isaac will become uncontrollable, at such a moment, so as not to die, you can use the first-aid kit with the shortcuts, sometimes it saves. These actions will have to be done on both sides After coping with this task, we take away the converter.Now your way lies to the crew deck.

Watching an epic scene ...

I will not spoil = P ah, okay, here's a screenshot for you:

Hint: this Brutus has about x2 HP, but his shoulders and torso are now much better protected, consider him your mini-boss of this chapter =) Well, in general, you somehow kill him there, and for now I'll write a sequel further (he-he-he).

As you kill the annoying monster, watch out for the Diamond Guide. Use the locator, run to the elevator, power it by inserting the battery. We quickly run away from Vaylor. We kill the monster. Here we are at the transport station.

Chapter 10: The Last Days

There are two news for you - good and bad. The good news is that Kendra knows where the shuttle is and is generally ready to launch it, the bad news is that someone has taken the navigation maps off the shuttle. Now you have to find them on the deck. In this chapter, I advise you to use the locator more often.

When you get into the elevator and go up to the upper floor, a cutscene will await you, where the mad doctor kills the survivor as a sacrifice. Unfortunately, you still cannot get to the doctor, the entrance to this room is located a little further along the corridor, to the right. In the room you will find some supplies and you can take the elevator downstairs.

A little further, in the corner, near a woman who has grown into the wall, in a box you will find what you are looking for. Be careful - the woman is not completely dead yet. To continue the passage, return to the room where the murder took place, and from there into the corridor. Move forward using the locator.

There are many battles ahead of you =) fighting off enemies, you will find yourself in a room with gravity and many monsters. In addition to the necromorphs, the room contains cylinders for replenishing oxygen. Use a map to get enough air. Search the adjacent rooms. Walk down the corridor and find a navigation map.

When you get to the elevator and go down, you will again find yourself in narrow corridors, and along the road on the right you can find a map. I draw your attention to this scheme, as it will allow you to buy a fifth level protective suit in the store. Further on earth you will find a text message with detailed description a new mini-game - Z-ball - we will tell about it a little later. In the meantime, open the door and you will find yourself in an unusual game room, where there is no gravity. Before starting the game, pick up the navigation map in the middle of the hall and deal with the necromorphs. The essence of the game is quite simple. You need to jump on the igniting platforms, when you touch them, they will go out, after you touch the last platform, catch a kind of basketball with kinesis and throw it into the hole. And so several times. Limited time. The game has five levels. There is a prize for each level passed.

After the game, using the locator, we move forward, killing everyone in our path))

After reaching the elevator, which will be in the room, go down to the large hall. Here you will find a solid battle with monsters. Explore all the rooms in the hall - you will find many interesting things, including a personal audio recording of the captain. Then we pass through the large doors in the middle. You can get straight to the shuttle, set up navigation maps, visit the next room and try to start the shuttle's engines.

You will meet again with the immortal necromorph. It's time to end this. Lure the creature to the shuttle engines, shoot off its legs, freeze it with stasis and run into the room next to it. Run engine check. And make a shish kebab out of it)

You will see Dr. Kane again, he will sit in the shuttle and ask you to release the shuttle clamps - which you should do. After that, you are left with the last passage of the tenth chapter. In place of the immortal monster you fried, pick up the power unit, explore the deck and collect ammunition. That's it, now you can go back (use the map and the locator), on the way, killing the necromorph returning to life and his new victim - the insane doctor. Return to the transport station.

Chapter 11: Alternative Solutions

Welcome to the penultimate chapter of the game!

Dr. Kane will contact you to explain the essence of the task - you need to find the cargo hold, find the marker in it and use the freight elevator to deliver it to the hangar. In the transport section, you will find a workbench and a shop. To make it as easy as possible at this stage, use a locator and a map, however, it is easy to find your way without using them. Soon you will find yourself in a large cargo compartment, having dealt with the necromorphs, reach the console and activate it, after a few seconds a marker will appear in your eyes. You can move it with the help of kinesis, but they will greatly interfere with you, guess who? ) That's right - the necromorphs, and then the tentacles behind the tentacles. In between attacks on the conveyor belt, pull the marker towards the lift on the opposite side. As soon as the marker is on the platform, it will be lifted into the landing bay, where you need to move too. The lack of a marker is clearly not to the taste of numerous local inhabitants, so the way back will be hard to call an easy walk.

Take the elevator up one level, then through the doors to the corridor and to the transport station. Here Kendra will restore the power supply to the elevator, and you can get into it (a new door at the transport station will become available). Take a run, take your time, check every corner ;) Then return to the corridor again and go to the large door. You will find yourself in a room with a save point, here you will find an unpleasant surprise in the form of a sudden quarantine and enemy attacks. Return to the room that was quarantined and go through the large door to find yourself in the hangar. Soon Dr. Kane will fly into it on a shuttle.

The automatic loading system does not work, so you have to turn off gravity and manually load the marker. Use the locator to get to the control room. There you need to drag the marker using kinesis. When you get to the arrow, switch it on the panel, put the marker on the perpendicular rails, switch the arrow again and lead it further to the shuttle.

Necromorphs will interfere with you in every possible way. On the second arrow, you will need to pull the marker directly under the shuttle, after which you need to restore gravity. Get to the shuttle along the corridors. Only a few seconds will not be enough for you - Dr. Kane will fall from the shot, and the traitor, I will not spoil you, will hide on the shuttle. The next communication session will clarify everything, including the origin of the marker.

And Kendra will turn out to be a government agent who just needed that marker.

You were left to die here. Or not?! Suddenly, Nicole will get in touch - your beloved, the one for whom you flew in. Glitches? Isaac is on his way to the flight control room. As we already got used to, we run back, killing everything in our path, or maybe we'll pick up something else that we missed at first (I'm talking about resources). When you find Nicole, she will give you the "good" news ... You can control the shuttle remotely. Kendra will have time to eject in a lifeboat. Leave the control room. Go to the landing platform.

Chapter 12: Dead Space

This final chapter begins on the surface of the planet. Exit the ship, Nicole will tell you to use the forklift to get the marker from the shuttle. Take a close look, there is a small mobile platform nearby - a loader, it needs to be pulled up with the help of kinesis to the ship. You can only pull the platform up to the side of the ship, there will be a control panel nearby - you need to activate it, the obelisk will automatically sink and, now with the help of kinesis, you will need to pull the obelisk together with the loader.

Hint - some bridges will need to be frozen.

To go further, you need to find a battery. We go to the only open doors, if necessary, we use the workbench and the store, go further and kill a couple of monsters, take the battery and drag it to the closed door. Drag the marker. It will be fun here and the smell of gunpowder hovers….

You will have to run a lot, just use the locator. Complete the mission (turn on the fan) Cool to run through the huge ventilation right? =) We go back and still drag the marker further. After the next buffer room you will find yourself on the street. The landscape is simply unreal.

Drag the obelisk to the edge. When you drag the obelisk further, a surprise awaits you ^^ A sea of ​​action =) You not only need to drag the marker, but also fight off tentacles and necromorphs =) Well, I hope you have coped with this task. Now Isaac needs to make legs from this planet. By the way, here you will have another surprise in the form of Kendra O_O She's back. And while you are in the room, you can watch the FULL message from Nicole. Unfortunately, your beloved is no longer there, and what you saw was only a figment of the imagination, supported by the mystical powers of the marker.

Fun fact - If you add the first letters of the Dead Space chapters to English language then we get the phrase: Nicole is dead - Nicole is dead

Use the locator and get to the shuttle, a small scene (I advise you to turn it up and turn off the light ^^ You will see the death of Kendra and immediately get acquainted with the main boss of the whole game, and now estimate how many first-aid kits do you have? How many cartridges? ))) And his name Hive mind.

BUGAGAGAGAGAGA

Okay, I believe in you ;)

The Hive Mind is the main boss of the game, the head of all necromorphs. He did not leave the planet, but telepathically controlled all the necromorphs. It is this creature that is responsible for all the misfortunes in the game, and it is her that you will have to kill if you want to complete the passage of Dead Space. The hive mind is more of a worm-like creature. Around the mouth, if you can call it that, there are yellow growths. The same growths are located in the chest, but at first they are closed by the likeness of ribs. To destroy the creature, you must first destroy the yellow growths around the mouth, and then, when the ribs "open" - the yellow growths on the chest.

You can choose any weapon, but I advise you to take something smart, for example RIFLE. If there are few cartridges on it, here are the alternatives: plasma cutter, power cutter, pipe cutter. You should attack when the creature "howls" at you, coming as close as possible.

Don't be afraid when she grabs you, keep playing Mad Max !! =) Shoot and don't stop for a second - that's the whole guarantee of victory. Naturally, it is worth dodging the blows of the tentacles. Also, to deal damage to yellow growths, you can use the exploding canisters, carefully left by the developers. Be prepared for the hive mind to spit out a couple of necromorphs - they will also need to be dealt with. Then there will be an attack with exploding balls, dodge. When the last growth is blown up, you will finally see how the boss, because of which your beloved died, dies. You've got your revenge.

ALL! You are great, and now run to the ship. This is the end of a great game Dead space .

Dead Walkthrough Space- The Ishimura spacecraft is not communicating. Equipment breakdown suspected. Then a group of specialists is sent to this ship to fix the breakdowns.


Team Lineup Isaac Clarke, Kendra & Hammond. The game begins with Isaac watching a video of his girlfriend asking for forgiveness from him. She works just at the Ishimura space station and in her video she also says that there are some serious problems on the ship, although she does not specify which ones. This is what happened to be found out.

Chapter 1. Arrival.

The crew tries to land at Ishimuru Station, but their ship gets hit by a meteorite shard and they crash. Fall on this station, and then the game begins. After leaving your ship, you first need to follow the team, following all the tips that will appear periodically. Upon reaching the double doors, Team Leader Hammond will tell you to open these doors. Then immediately go to the computer, right to the wall itself and to the right, there will be a door, open it. Look in this room. You will find an illuminated touchscreen control panel. After listening to the dialogues of the characters in the next room, turn on the computer (button E).
After switching on, you will hear some noise. Suddenly the lights go out, two monsters break through the ventilation grill, jump from the ceiling and immediately attack two crew members. The girl will shout, "Get out of there, Clark." Better to follow her advice. The door, which is located on the opposite wall from the computer, is initially locked, but if you wait a few seconds, one of the crew members manages to unlock it. We go through this door along the corridor, do not stop and do not turn around, a monster will immediately run behind you, and you have no weapons yet. To speed up, hold down the SHIFT button. We run into the elevator, quickly press the button and go. True, the monster will try to open the elevator doors to attack you, but the doors will close, flattening it. Having arrived at the desired floor, examine the room. Break the boxes (SPACE button) Dead Space walkthrough, there you will find credits, patrons. Take the pistol from the table. To view your inventory, press the I button. Next, find the door, to open it, break the door's power supply. After that, a monster immediately appears, which attacks and kills some person, and then this monster immediately attacks you. To kill a monster in any case, do not shoot at the torso, it is useless, the most effective way is to shoot off the limbs. Monsters are even able to live without a head in such a way that a standard headshot does not always help. During the passage of the dead space, you will periodically come across audio and video recordings.

To start searching

There may be useful tips... After passing the corridor, you will find yourself in a room where the commander will contact you, and you will receive a task to repair the transport system. You need to find a dashboard for your computer. In this room you can save yourself and search lockers looking for useful items. We go further, next to the lockers there is a door through which you need to go. You have two ways to go right or left. To the left there lies a dead monster, which will immediately come to life when you approach it, so it is advisable to bang it while it lies on the ground, shooting off its limbs. You can also save there, and find an audio recording where the Ishimura worker will say that you can kill monsters by shooting their limbs. If you go to the right there you can get a stasis module that can slow down objects around and monsters. After passing the corridors, you will go out into the hall where the trolley is located, in the same place the jammed door periodically closes and opens. It will not be possible to slip through in any way. Use the stasis module to slow down the door. After passing the jammed door, you will reach the next save. Use the B button to find out where to go next. But before continuing the path from the preservation, turn right, there will be men's and women's toilets, search them, unless of course you are not afraid of the darkness and the noise of falling pieces of iron. There are no monsters, but it is very dark. After some time, a light will start in the toilet, and you will find cartridges from a plasma pistol there. To the left of the save will be a corridor where you can find a box with cartridges. Next, press the "B" button and follow where the locator paves the way. We go into the room, here you will find a special node that will help you in the future to improve weapons and abuse. The minecart needs to be replaced here. Use the stasis module to slow down the two mechanical claws of the robot.

Be careful, two monsters will appear in this room after a while. After manipulating the robot's pincers and stasis module, the minecart will be replaced. If you stand facing the trolley, a monster will immediately appear on the left, quickly kill him. Now is the time to go back. Use locator B to plot a path. Go through the jammed door again. Better to immediately reload the weapon (R button). Because two necromorphs will attack you at once.
clue Dead Space walkthrough: Search the corpses of monsters, there may be ammo or credits.
Your next task is to find the dashboard. To do this, use the locator and go to the elevator. You can save along the way. When you get into the room, look at both. Two monsters will immediately trample on both sides, killing them, another one will immediately come out of the ventilation shaft. After a while, the last monster will appear. In total, you need to fill up four monsters. Go to the next elevator, using the locator, on the way, kill the monster that will appear near the elevator.

Take the elevator to the top floor. Without leaving the elevator, take a closer look, there will be two corpses nearby, one of them is a monster. Do not come close, otherwise it will come to life, kill him while he is lying on the ground. Go along the deck, there you will find the key to the engineering compartment. Be careful, after a while a necromorph will appear. Having received the key to the engineering compartment, you need to return to the elevator. A monster will appear near the elevator. Kill him and take the elevator downstairs. Keep the cannon ready (with the right mouse button). As soon as the doors open, you will be immediately attacked. Using the locator, we get to the engineering compartment. Press “E” to open it. The key you found earlier will be used. There you will find a workbench, thanks to which you can improve your weapons with the help of power nodes.

We take the information panel from the table and go back. If you forgot your way back, use the locator (button B). When you exit the engineering compartment, you will be attacked by two necromorphs from both sides.
clue Dead Space walkthrough: Many monsters are very cunning and can pretend to be dead. They come alive only when you walk by. If it seems to you that you have not killed this monster before, then kill it now, while it lies on the ground.
You need to get to the elevator, on the way you will meet a monster pretending to be dead, kill him and move on.
Take the elevator upstairs. Be ready, a monster will run out at high speed on the top floor and try to kill you. Having dealt with him, we get to the trolley control room. Use the dashboard on your computer as shown in the illustration.


Call the minecart using another computer nearby. The next task is to get to the ship on which you arrived. Using the locator (button B) we reach the elevator, it will take you to the boarding terminal. After passing along the corridor, you will see a monster that will hide around the corner. When chasing a monster, be on the lookout because another one will fall on your head, kill both and move on, so you will get to the boarding terminal. After passing the landing terminal, follow to the flight hangar. There a tailed monster will attack you. Kill him and enter the ship. The computer will say that alien life forms have been detected and after that everything will start to explode. You quickly run out of the ship. The ship explodes and falls down. Four necromorphs will attack you. They will be far enough away so it won't be difficult to kill them.
Since Calleon's spaceship is destroyed, there is no choice but to try to figure out what is happening here. We need to get to the medical deck and find the captain's XI, maybe he will give some clarification. To get there you need to get to the trolley. Again we go to the boarding terminal, from there we turn left to the “Transport station”. For convenience, use locator B.
clue Dead Space walkthrough: On the way to the transport station, turn right into the men's and women's restrooms. There you will find a first aid kit.
Use the elevator to get to the transport station. There is a store where you can buy: weapons, cartridges, power nodes, stasis batteries, armor and a first aid kit, as well as a workbench where you can improve all this.


Each weapon has an additional attack, to find out which one to aim and press the spacebar.
Hint: At the beginning, only the second level armor is available. Therefore, if you do not buy armor at all, but only buy at the end of the game of level 5, you can significantly save money (I thus played the game on an inhuman level of difficulty. I spent the money saved exclusively on ammunition and on improving weapons. At a high level of difficulty, monsters are very tenacious .)
Now get into the cart and drive. End of the first mission.

Go to Main page

If you have problems with the passage of the game Dead Space, you can always use our tips and information for action. We describe in detail the steps that need to be taken to complete the game. Dead space... In the most difficult places, we add pictures that can help you. Walkthrough Grandfather Space read on our website.

Arrival

Watch the video, listen to your comrades. Open the door on the right, examine the console, and then when a monster attacks you, run to the elevator. Take the pistol, shoot it at the control panel. Go through the broken door and then pick up the stasis. Slow down the door with it, go through it. In the hall, press first on the right lever, then on the left. Shoot the minecart with stasis. Click on the middle panel. Kill in the marked area, eliminate several monsters.

Intensive therapy

Get the item from the dying lady, now you can lift heavy items. Move the boxes, and then follow the previously laid path, on which you will have to kill the next batch of monsters. When a creature with tentacles attacks you, press E, and eliminate the cocoon with the F key. Place the electric block in the wall, go up using the elevator. Next, aim at the left part, press F. Walk quickly through the area without air, go to the panel, jump over and insert another energy block into the socket. Get the bomb, go back, detonate the bomb. After a short cutscene, eliminate the monsters, go after the captain's corpse. Receive access codes, continue your way at the sign.

Orbital correction

Reach the target, use kinesis to lower the lever down. Go downstairs, pull the lever towards you and use it to pass the abyss. Lower one more lever, go further, disinfect yourself, and then eliminate the crowd of monsters that have appeared. Now you need to hook two energy blocks using stasis, and then insert them using kinesis into the central mechanism. Go into the elevator, go downstairs so that the blades don't kill you - go right. Get to the doors that will allow you to go out into outer space. Fly to the required platform, start the engines and deal with another crowd of monsters. Try to save ammo, after the victory, go to the minecart.

Death is inevitable

Exit to the bridge, you will see huge monster in armor. To win, go around the monster and shoot in the back. After the victory, go through the gas fountains, then you will again have to fight another huge monster (also shoot in the back). Then shoot stasis at the wires, activate the panel. Go back, go downstairs, activate another panel, and then follow to the third floor (there you also need to activate the control panel). Now go into outer space, beware of meteorites. Destroy them with a huge cannon, and then go back and deal with the monsters.

Deadly addiction

Follow to the room that is marked on the radar. In it you will find the ingredients you need, but you will also find a surviving person who will wake up another monster. You cannot defeat him, because the creature knows how to grow limbs, so just run to safety. Reach the large hall, while your partner is trying to open the door, you will have to fight the monsters. Go through the opened door, take DNA samples. Collect oxygen and move through the locations, you need to get to the room where you met the surviving scientist. He will set a familiar monster on you. Cut off his limbs, and then freeze it using a mechanism that can be activated in the place where the professor stood. After the victory, go to the transport station.

Dangerous impurities

Meet with your partner. On the map you will have marks, you need to get to each and kill the monster (lets gas, so be careful and don't get close). Go back, release the gas into outer space, go to the food compartment and start a fight with the boss. Shoot his limbs (marked yellow). When the gate opens, shoot the monster with all the items at hand. After defeating, head back from where the chapter started.

Step into the void

Go down the elevator, drag the four debris under the powerful energy beam. Turn off gravity, fly further, and pass the abyss with the help of a minecarriage. While your girl will open the door, you will have to fight off the crowd of monsters. Follow the elevator to compartment C, take the electrical block, insert it into the opening to activate another elevator. Follow it further along the route, as a result you will find yourself in a large hall. Shoot stasis at the gravity grapple, and then one of the flasks. Slide through the half-ring into outer space. Destroy the captures, place beacons on the asteroid. Go back to the elevator, go upstairs, start the asteroid.

Find and rescue

Exit the transport station, sit in the minecart and wait until it takes you to the right place. Collect all antennas, insert into the center mechanism. Go up to the bridge, get a text message. Get to the huge cannon, start controlling it. Shoot the yellow tentacles. Then head to your new destination.

Dead on arrival

Go to the huge hall, break the flasks. Throw out all the green balls from the ship, large boxes can be pushed back with kinesis. Take the missing battery near the elevator. Go to the next large hall and again fight off the crowd of monsters. After the victory, continue moving, there will be fire on the sides. Use kinesis to destroy the glowing blocks. At the end, fight a familiar monster that can be killed with shots in the back.

Last days

In this mission, you need to find three keys. Go to the playroom, study the rules of the game (they can be found in the text message). Play ZI-Ball and get three of your keys. Go to the shuttle, try to start the engine, in response only monsters will appear. Kill them. Move the limbs of the largest to stasis (opposite the shuttle turbine). Try to start the engine again.

Alternative solutions

Proceed to the Hall with the Obelisk. Activate the panel, kill the monsters that appear, and then move the Obelisk to another part of the room. Go ahead (be careful, a crowd of monsters can attack you at any moment). Having parked the shuttle, go back and move the Obelisk, it needs to be delivered under the shuttle. Chat with Nicole and then hop on the shuttle.

Dead space

Drag the loader to the shuttle and then activate the control panel. Take the loader back, the door will be closed, so go to the door opposite. Take the battery, insert it into the wall, which is near the closed gate. Keep moving the Obelisk, restore energy. Click on the panel again, drag the Obelisk to the podium. Now go to the room on the right, treat yourself and the costume, talk to your friend. Go to the deck, get ready for a boss fight. Shoot off the limbs on his head, and then on the body. After the victory, get into the lifeboat, leave the area. Completed the game.

Dead space is another masterpiece from EA Redwood Shores. This game is quite impressive. It has all the qualities for Action / Adventure games: a wide variety of monsters, locations. One drawback - few weapons. I do not recommend playing for the faint of heart.

Arrival

The game begins like your technical ship, for some unknown reason, makes an emergency landing on another ship. But there are no souls on it. What happened here? Now you and your crew must find out what happened here. Follow the other crew members. To get on the ship, you need to open the door, go to it and click. Now you need to activate the elevator. Click and follow the blue strip. In the honor of the room there is a computer, go up and click. Wait a minute, and suddenly, out of nowhere, monsters jump out. Two of your crew members are killed, and you yourself must flee. Otherwise you will be eaten. Run along the corridor, bloodthirsty monsters are chasing you. What to do, how to be? And suddenly, like a miracle, there is an elevator at the end of the corridor. Run in there.

You go downstairs, you find yourself in the room. The first weapon is on the table next to the corpse. Now be careful. You need to open the door. To do this, you need to destroy the device near Tver. Opple. The first monster. My advice to you is to shoot not at the body of the monster, but at the extremities, and then finish off in close combat. Save ammo. You go further into the distance of the corridor. Be calm, you will not meet 1 monster yet.

You get to the computer. Talk to the surviving team members. You receive your first assignment. It is necessary to replace the broken carriage with a new one. So that the rest of the crew can get to the bridge. We leave along the blue strip or you can follow the map. You see a broken door ahead. It opens and closes quickly. There is ammunition next to the door. This is a stasis battery. Aim at the door and click. The door slowed down. We seize the moment and run. Moving on. We go into the room. This is the room we need. You need to replace the broken part of the carriage. To do this, you need to activate three remotes. First close to you, then the farthest.

The pistons will extend. But one is not working properly. You need to slow down time again. We run to activate the third remote control. That's it, the first mission is complete.

Now you need to activate a new mine cart (transport) from the control room. We leave to the computer of the command center. After going there, it turns out that the computer is not working. Need a new board. We leave for those compartment. Follow the blue strip or map. But now be on the alert. Monsters will attack you more and more often. You get to the elevator. You go down to those compartment. This is the scariest part of this chapter. Take the elevator up to the next level. There is a card that will open the door to the control room of those compartment. Now feel free to go straight there. In this compartment you will find a board and there is also a device for improving your weapon. You can decrease the reload time or increase the power of your weapon and much more. Now, having taken the board, get out of those compartments. Through the elevator. We return to the damaged computer. Replace the part and activate the minecart. Your crew is sent to the bridge, and you must return to your ship and prepare it for departure. We kill all the monsters along the way.

After reaching the ship, go into it and activate the main computer. What's going on? It starts to explode. We get out of the court. With difficulty, having managed to jump out, the ship explodes. After the explosion, monsters will attack us wave after wave. Everything. The ship was your hope to get away, but it was blown up. What's next?

You must find the corpse of the captain of this ship infested with monsters. He must have access to the on-board computer. Your Remaining Crew found out that he is in the morgue. You must use the trolley to get into the honey compartment. Follow the blue line to the mine cart. Come into it.

Intensive therapy

The second chapter begins with the discovery of a woman. Apparently she survived, but not for long, before she has time to give you kinesis, she dies. You need to get over the fence, use kinesis. Aim at the fence and click. We go further, focusing on the map. We reach a room with three doors. You can go through two, but the third is welded. And to get through you have to blow it up. You need to collect explosives.

Go to the Medical Compartment. This is a left-hand door. The fun is about to begin. You are attacked from all sides. The quarantine mode has started. And some doors closed. And the door that we need closed closed. To remove the mode, you must kill all the monsters. Be careful, before the monsters fell on the sides, from the front. Now you will be attacked from all sides. We go further and lead a terrible picture. The man behind the glass kills, a completely new kind of monster. You must avenge the poor fellow. Kill the monster. Following the map, move on. And so we have reached the first goal. In the main office, the medical compartment, lies. After taking it, return to the room with three doors.

Everything, there is, picks up hydrocyrin. Go to the right-hand door. Finding your way around the map, you should go to a large room, in the center of which is a huge incubator. It must be moved to pass, use kinesis. We reach the elevator. It does not work, you need a battery, it lies nearby, and again kinesis comes to the rescue. We rise to a higher level. There is a cliff, in order to cross it is necessary to move the incubator to the place of the cliff with kinesis. And we continue to walk, focusing on the map. We open the door and what do we see? This is the door to outer space. But don't worry, your suit will save the day. We pass this place, open the door. The entire room was sealed. We open the next door, and you find yourself in one of the gravity generator. You are standing on a platform. But you have to go to the other side. And the distance is not so small. What need to do? You should have noticed the gravity control panel. Disconnect it, and boldly jump over. We open the door and see hydrocyrin on the table. We take it and return to the room with three doors.

Now having two necessary ingredients, we collect explosives and mine barricades. We open the door and go through. On the way, we see very strange things as a woman kills a man in masach and then slits her own throat. We go into the morgue and see how some creature makes a monster from a human corpse. Now we understand where they come from. Naturally, we must not let him go. We kill him. There is an access disk next to the corpse. We take and go to the trolley.

Orbital correction

You have received an order to put the ship's gravitational field in order. Go to the engine room. As always, be guided by the map. Don't forget to pick up the right items. You are now in a large room, divided into da sectors. There is an abyss in the middle. You need to activate two levers. One is on one side (Where are you), the other is on the opposite. Follow the map to the first lever. After finding it, lower it using kinesis. How to activate another? What to do? As you should have noticed, there is a movable platform between the sectors. She will help you. But be on the alert, before you have time to land in the second sector, all sorts of monsters will attack you in droves. Activating the lever, return the same road to the elevator to go down to the lower levels.

Now your task is to turn on gravity. Walk forward a long time using the map. And finally, here it is, the gravitational device. It is a huge body. And on the sides there are two coils. We must connect them. But they are spinning fast. Use item stasis and kinesis. All of you have normalized gravity.

Now you need to start the engines yourself. Follow the route. Find your computer, activate it. Everything. This chapter is over. Return to the minecarts.

Death is inevitable

In this chapter, your main goal is to enable ship defense. You need to find the captain. He's on the bridge. Guided by the map, you can easily find it. You now have an assignment. You need to activate your computer. Follow the map. On the way, you will meet a new monster, which is not so easy to kill. The machine gun, which is sold in the store, should help you. Find a computer behind the monster in the room.

Activating you need to take the elevator down to the level below. There is a video in the elevator that will help you figure out what happened here. Again, walk long and hard, guided by the map. Since there was an accident in one of the compartments, some wires were damaged, and the ship was simply wasting energy. Your job is to turn off the power supply. On the way to the lever, you have to kill a lot of monsters, so be on the alert. And so we got there. As you can see, the wires are bare. They block your passage. Use stasis. After activating the computer, go back.

Now your task is to redirect the energy towards the cannons. There is another elevator from the elevator on the other side, we go down to the first level. There we activate the computer. And we return to the elevator.

We rise to the third level. Now it is necessary, using cannons, to fire at the asteroids until the automatic defense is activated. We open the doors and run to the next elevator. Now you have to go into outer space. But this is still nothing, in comparison, that you will be constantly under fire from asteroids. You will need to make your way slowly, from cover to cover. After going there, you will have a small mini-game. It's pretty lightweight. Just shoot the asteroids until the automatic defense system kicks in. After that, return to the minecart.

Deadly haven

In this chapter, you need to create an effective poison against monsters. To do this, run to the laboratory. Using the map, we leave, the already familiar room with three doors. From there follow the blue line or the map and move into the laboratory. The path is not close, but since you've already been here, I think you can handle it. Having reached the room, in the center of which there is a flask. In nutria, which is an ingredient for creating poison. Wow. They did not expect? Another survivor. But as it turned out, he was slightly touched. He believes that everything that happens here is God's plan and sets a new kind of monster on you. You cannot kill this monster. He has an increased regenerative ability. You just have to run

Run back to the room with three doors. We run through the door that you opened the door by blowing up the barricades in the second chapter? So kat you have a flask with an ingredient. You need to add some of these monsters' fabric. You were in this part of the deck in the second chapter, so you can easily find that compartment. Will help you with these cards. We find a room, in the center of which there is a device, we insert a flask there. All fabric is added.

Now again you need to get into the room in which you took this flask. Run through the room with three doors to the room where you first met the Crazy Survivor.

After you've added the last ingredient, this survivor invites you to a conversation. And will open the door. You should go and find out what he's up to. He says he wants to transfer this virus to earth. Then he leaves. Leaving an invulnerable monster on you. You should have noticed a freezing system in the center of the room. From this it follows that in order to kill a monster, you need to freeze it. After all, then his tissues will not be able to regenerate. Drive the monster into this device and immobilize it. While he is regenerating, run into the room and activate the remote. He's all dead, go to the trolleys.

Dangerous impurities

After passing through the central room of this deck, you meet Hamond, who informs you that mutated people, secrete poison, thereby poisoning the atmosphere. You must find and destroy everyone. Following the map, you will exit to the western greenhouse. Monsters like mosquitoes will attack you. After killing everyone, replace the feed tank. To do this, activate your computer. This greenhouse has two rooms that contain these venom-producing creatures. Visit them and destroy the monsters. Next to the control panel for the activation of the feed tank, the elevator will turn on. It will lubricate you to lift you three levels up. But you first need to go to the second. There is another room with a poison-secreting monster. Now you have a long way to go to the next monster. By following the map, you can easily find it. You will have to go through a compartment in which gravity is turned off, as well as through corridors that are teeming with monsters. As a result, you will find him and kill him.

Now head back to the central room of this deck. There are two greenhouses on this deck: east, west. You visited the western greenhouse, now from the central room, guided by the map, go to the east.

Once there on you, it immediately runs over big monster... Kill him. Here you need to do the same as in the western greenhouse. Run around all the rooms, kill all the poison emitting monsters. One monster is in the room where the feed tanks are. Use Stasis to kill him. After his death, replace the feed tank. As in that greenhouse, take the elevator to the second level. Again a long way to the next monster. Follow the map. After killing him, return back to the central room.

It is necessary to turn on the ventilation system. There is a remote control in the center of this room, activate it. A large door will open, we pass there. There will be a hole at the end of the corridor. We put poison into it. The poison had a bad effect. You must destroy the creature that poisons the air on the ship yourself. Open the door and get started. This monster has weak spots on its tentacles. Aim at them. And when he spits you out large exploding stones, shoot them. It is quite easy to destroy it. Then follow to the cart.

Step into the void

In this chapter, you must send a signal to the rescue ship. To do this, you must attach a beacon to an asteroid and throw it into outer space. The lighthouse is located in the engineering compartment. Run from the minecar to the center room of this deck. There will be an elevator. He can move down three levels (A, B, C, D). But you need to level first. From the elevator, follow the map. You find yourself in a room with disabled gravity. It is necessary to turn on gravity, but for this it is necessary to destroy all the asteroids. Use kinesis. After destruction, activate the remote control. Then we run to the control room of this sector. There is a key on the floor next to the corpse. Taking the key, we run to the elevator and go down to the level.

From the elevator, run across the map until we find a moving platform. She will help you get to the other side. On the way, monsters will fire at you. Wow, is this a hallucination? This is Isaac's girlfriend. She will help you open the door. But you have to cover it with fire. While she tries to open the door. Then go into the room and take the lighthouse. We return to the elevator. We rise to the level.

Now you need to anchor the beacon to the asteroid. Walk guided by the map, you will come out to the elevator, which does not have enough battery. She is at the other end of the sector. Move it from there court. Taking the elevator. You will find yourself in a huge room with a large asteroid in the center. Attach a beacon to it and blow up the gravity locks. Now head back to the elevator with the battery. Move the battery back to another elevator. Pick up on it.

There you will see a closed door. So the card came in handy. Open it. There will be an asteroid launcher. Run. Everyone now has to wait until they hear us. Return to the minecart.

Find and rescue

In this chapter, you will need to fix the bullets to transmit the signal. To be heard by the rescue ship. To do this, run to the bridge. There is an elevator, go up one level. But before that, you need to remove the quarantine mode. Those. kill all monsters. From there, follow the map to the moving platform. Walk forward until you reach a large antenna.

You need to make sure that all working antennas hit in the first row. And then return to the control room to transmit the coordinates. Hurray received a signal from the rescuers. Now they will fly for you. But rescuers found a lifeboat in which to find one of the creatures. You must warn them. But some creature has attached itself to the antennas. You must destroy it. You need to get to the cannon, be guided by the map.

Having reached, sit down in the cannons and shoot at the yellow parts of its tentacles. Destroying one tentacle after another. Hurray, the creature can no longer be attached to the ship's hull. The antenna is free. After contacting the ship, we find out that they opened the capsule. This means that they are all dead. And their ship collides with these. But it is not all that bad. We can now talk to Hamond again. He found the shuttle. On which we can fly away. But you need to prepare it for departure, go to the trolleys, and then to the technical compartment.

Dead on arrival

In this chapter, you need to find a converter. Walk away from the cart using the map. You find yourself in a room with disabled gravity. To go further you need to throw green spheres that fly in this room into space. There is a huge hatch on the floor in the compartment; to open it, break the eclectic generators next to it, there are six of them. Then, using kinesis, throw all the orbs into space. You can now leave this sector. Get to the elevator. But, as we can see, there was an overload of the batteries. To start the elevator again, you need to move the battery located on the other side of the room. All elevators are on.

Another survivor will contact you in the elevator. He will report that the monsters appeared due to the fact that they took away the obelisk from the planet. And he says that they must return the obelisk. Go further from the elevator, guided by the map, reach the store. You can now buy new weapons. Then move on.

You are trapped in a compartment in which fire is erupting from the turbines. You need to de-energize the turbine. To do this, destroy six generators. After that, go to the turbine and take the converter. All there was a critical breakdown in the technical compartment. We must get out of the court. Run to the cart.

Last days

Now another problem is who took out the shuttle navigation maps. They are scattered all over the deck. You must find them. Follow the map, you get to the bar. And you see how, behind the glass, a crazy believer kills one of the survivors for the sake of faith. Go there. Take the elevator down one level. There, in the box, you will find the first board.

Now go to the exit from the bar. There is a closed door, but we have a key to it. We open and go in. After that, you find yourself in a room. There is a closed door; to open it, you need to find a lever. Find the lever using the map. After finding the lever, activate it and return to the door. Pass through it, you find yourself in a sector in which the air conditioning system is violated. You will find another board here. Use the map. There is little time to search, there is no oxygen. As you find the current, leave immediately.

Now head back to the elevator. Another fee is in the gym, a map will help you get there. Taking it, go to the elevator, which you should have noticed on the way to the court.

Going up one level, you find yourself in a room with beds. There is another board at the end of the room. But we must somehow get to her. Or rather, you need to move the beds so as to go further. Taking it, go back. You have collected all the boards, now you can talk to the survivor who told you about the obelisk. Progress by following the map. You will go out into the room, behind the glass of which you will find him. After the conversation, he will open the elevator for you.

You should now go to the shuttle and prepare it for departure. Fix it. Do a test run. And wait for your new friend to pass away. He will bring the shuttle to the mooring deck with the obelisk. Detach the shuttle. And go to the trolleys.

Alternative solution

There is an obelisk in the cargo hold. You must lower him into the shuttle. Go down the elevator to the cargo hold. Find the remote control there and pick up the obelisk. And then use kinesis to transfer it to the other side. Then return to the cart and go to the elevator door.

Take the elevator. Go, guided by the map, sectors with a shuttle. Go to the Docking Deck. There will be a ship waiting for you. You will have to. Place the obelisk on the shuttle yourself. Turn off gravity and get started.

It's pretty easy. So I think you can handle it yourself. After loading, go to the ship. What's going on? Your new friend has been killed. This is the betrayal of the girl who helped you all this time. What will we do?

Again, Isaac's girlfriend contacted you. She asks you to go up to the control room. There, activate the computer and the shuttle will begin to return. But someone left the shuttle using the capsule. You probably guessed who it is. Now go to the shuttle.

Dead space

You flew to the colony. Now you must put the obelisk back in place. But first, unload it. After that, you will have to move it to the right place for a long time and persistently. On the way, different monsters will attack you every now and then. There will be many obstacles. For example, there it will be necessary to raise a bridge, and at this time to have time to build an obelisk. But the bridges are sinking fast. In general, in this mission you need all the skills acquired during the game.

Having reached the end, you need to put the obelisk in its original place. You put it on. And here:. In general, see for yourself. You will need to get out of here quickly. Run towards the disarming device. After you have gone through this process, you again come across this woman who is your limit. This time she wants to take the obelisk with her. You must stop her. Run as fast as possible through the tunnel to the shuttle. But no. She got ahead of you. What to do?

Out of nowhere, a tentacle appeared. They killed the woman and they want to kill you. A large, creepy monster appears following the tentacles. Shoot at the yellow parts of his body. After destroying it, run to the shuttle and that's it. Enjoy the video.

An unusual game with interesting ideas - from Electronic Arts? A year ago, I would have been very surprised if you had told me who about this. A game publisher that has produced dozens of sterile sports simulators, multiplied add-ons to the Sims without number and regularly fed the market with games on film licenses, was associated with anything but innovation.

However, as of the autumn-winter 2008 it turns out that way. It is a forerunner and generator of new ideas, unusual worlds and atypical games. Listen to it - "a sci-fi horror movie from Electronic Arts" ...

Nervous collapse

The plot in Dead Space is utterly banal. The huge spaceship "Ishimura" of the "Ripper of planets" class - it literally destroys planets in the process of mining - at one fine moment stopped communicating. To clarify the situation, a small repair shuttle is sent to the Ishimur.

On board, among other crew members, there is a certain Isaac Clarke, an ordinary engineer, completely unsuitable for the role of a rescuer. After a hard landing on the Ishimura, it quickly becomes clear that the radio silence and emptiness on the quay deck were caused not by malfunctions, but by terrible monsters that flooded the ship. The crew of the repair shuttle is partially eaten, and the survivors scatter to different angles, so that later by joint efforts, communicating mainly through radio communication, they damn ship.

Here is "Ishimura" - suspiciously silent. The Kellion crew does not yet know what awaits them in the near future.

Despite the standard start, the game develops quickly enough to full power. The merciless space is overboard, there is not a soul around, and Isaac, along with the player, gently picks up a feeling of loneliness and despondency, which only intensifies from rare radio communications. This is far from a bright future where the careless spaceships ply the vastness of the smiling universe. Even with the shimmering light and bloody corridors torn to pieces aside, the overall design of the ship and its surroundings sets the mood for autumn.

But it is not the empty corridors, the semi-darkness, the sudden attacks of monsters and traces of blood on the walls and floor that frighten us. The environment only sets the mood, the sounds strike the real blow. It is the strange and disgusting sounds, whether it be incomprehensible screams or crazy disgusting grinding, that make you nervous and look around. They do not escalate the situation by warning of future danger. No, it was too easy. Sounds hint that Ishimura is far from a safe place and should not be relaxed.

The developers of the sound, which is often supported by some unpleasant, but not fatal event, hint that something terrible is about to happen. Internally, you make yourself for the worst, but nothing happens. The charge of fear does not receive an emotional outburst, and after a while you begin to be afraid constantly, literally every minute of the game. This incredible psychological attack is bearing fruit.

When monsters appear, the release of emotions and adrenaline, especially in the early stages of the game, is so confusing that sometimes your hands naturally tremble. Voltage is a constant satellite of Dead Space; expectation and half-hints, and not the gaping jaws of monsters, make us look around nervously. At the same time, there is not a single safe place in the game. Even in rooms with save points, monsters sometimes live.

Cruel space

Fights with monsters also do not allow to relax. Necromorphs, as the alien life form is called, are very resistant to damage, and the only effective way to fight is to cut off important limbs. You have to use a plasma cutter and other construction tools to literally gut monsters.

You will have to periodically go out into space, where monsters are also found.

There are always not enough cartridges, you have to think over every shot so as not to be left with empty pockets in the midst of the battle. Saving is the cornerstone of the gameplay. And business is not limited to one cartridges and first aid kits. Weapons and equipment need to be improved, and power blocks - this is the material for improvements - are always in short supply. Therefore, you have to choose what you really need and where to invest in the first place. Spare parts are enough for a maximum of three models.

Unfortunately, the developers were unable to keep up with the hard pace for the rest of the game. Having overcome two-thirds of the levels, you will significantly improve your weapons, and the fights will become much easier. At the very end of the game, monsters will no longer be a problem at all, which somewhat dilutes the atmosphere of tension and fear. However, we have to deal not only with shooting necromorphs. Dead Space is full of unusual game situations and interesting puzzles.

The biggest surprise comes from spacewalks. There is nothing special about this, but how these situations are presented. Suddenly, all sounds disappear, and we hear only the pounding of Isaac's heart and deep breathing. Zero-gravity zones, where the walls and ceiling become the second floor, and the character dashingly jumps in any direction for tens of meters, were also excellently successful. True, in the last two levels, the game slides down to the banal extermination of monsters and persistent pushing of the cart across the entire map.

In the game, even the backpack is made in an unusual way - in the form of a holographic menu.

The picture in the game is surprisingly lively, although sometimes there are muddy textures. It can be seen that the designers and artists have worked on the overall style of the game. Isaac's smooth and unhurried movements, great animation during close fights with monsters and in interactive scenes. In Dead Space, the interface is built right into the game: the health indicator is a strip on the back, the number of cartridges is right on the weapon, the level map, the task menu, the backpack are displayed as a hologram and the game is not paused.

The plot, despite its seeming simplicity, will surprise you with unusual twists several times. But the scriptwriters, having taken a rather vigorous pace at the beginning of the game, sometimes too abruptly and pointlessly cut off storylines... Especially the characters got it. But the heroes in Dead Space can be counted on the fingers of one hand.

It was seen that the developers were trying hard over the universe of the game. Animated comics before the release, a full-length cartoon after, it even threatens to reach a feature film. But nevertheless, upon close attention, logical inconsistencies in the plot immediately become visible, and additional materials - such as the Dead Space Downfall cartoon - do not provide answers to questions, but only add new contradictions.

However, these discrepancies cause only surprise, but not irritation. The plot in Dead Space serves rather as a framing for surprisingly accurate and careful citation of primary sources, eerie atmosphere and verified gameplay... I would like to believe that the contradictions will be eliminated in other games in this universe. If such, of course, will appear ...



Despite a number of miscalculations in game mechanics and plot, Dead Space deserves close attention and genuine love / fear. The heart is captivated by familiar and carefully quoted elements from Aliens and Something (this, by the way, is already a film classic), an unusual approach to intimidation and an incredible atmosphere of panic, uncertainty and the future. Have you already made up your mind? Then get ready for an exciting journey.


ADVANTAGES LIMITATIONS
Fascination
9
battles with monsters, various game situations, an atmosphere of feartowards the end of the game, the fights become too easy, in the last levels game situations are too often repeated
Graphics
8
nice models, special effects, level design, animationat times muddy textures
Sound
10
sounds and music that skillfully create the atmosphereNo
Game world
8
at first glance, a rustic, but strong and with unexpected twists and turns storylogical inconsistencies in the plot, disregard for the characters
Convenience
9
management, optimizationsome bugs, it is inconvenient to play on non-widescreen monitors

Basic character skills

Isaac can move in two modes - walking and running. Except for speed, they are no different. It is clear that you need to run when you are saving from monsters, and walk - all the rest of the time. But the character can run sideways. You cannot shoot, but this way you can dodge shots, especially when the enemy is far away. The camera in Dead Space is directed in such a way that you can clearly see what is happening in front, and you can hardly see what is happening from the side. Therefore, turn your head more often so as not to miss the necromorph.

If you do not have time to get out of the vacuum before the oxygen runs out, exactly this will happen - Isaac will suffocate and die.

When you get into the zero gravity zone, the rules change a little. You can also walk and run, but now not only on the floor, but also on the ceiling with walls. To quickly move from wall to ceiling, aim at the place where you want to get, and press the jump key. Isaac will bounce off the surface and fly across the room.

Thus, you can simply move quickly or flee from monsters. If for some reason Isaac is unable to jump, the ammo indicator will blink red and you will hear a characteristic sound. Remember that monsters in zero gravity also crawl well on walls and ceilings, so you have to actively twist your head to determine their position and not miss the attack.

Sometimes you have to go into outer space. At the same time, an oxygen indicator appears on the character's back. When it's over, Isaac will die of suffocation. Oxygen will automatically be restored when you get to the room with air. You can also refuel from special devices or carry a spare air cylinder with you. Know, if you find a container with oxygen, it means soon to open space.

If the monster got too close and you do not have time to shoot, then you can hit it well with your fist. From this he will fly back, and you can shoot or run away. The punch does not deal serious damage, but it stuns the enemy. So only use it to push the necromorph away. If the monster is crawling on the floor, you can kick it. Thus, it is convenient to finish off wounded monsters and crush small monsters, having previously slowed them down with stasis.

If you get lost, use your navigator to find your way.

Sometimes the monster does not just strike with razor-sharp growths, but jumps on the character and begins to bite on different parts of the body. To reset the necromorph, press the key indicated on the screen. Sometimes Isaac not only knocks back the enemy, but immediately kills him without your help. This usually works against small monsters. However, such attacks are very harmful to health, so sticking monsters should be feared like fire.

And of course, the most important aspect in the game is the ability to handle weapons. It would seem, what can be difficult here? Aim and shoot. This is so, but at least at first you will have a shortage of ammo, so you need to shoot accurately in order to inflict maximum damage. Also, each weapon has an alternative firing mode, so sometimes you need to think about how to charge the necromorphs in a particular situation.

In Dead Space, level navigation is neatly done. You can open general map, study it in detail from different angles, increase or decrease the view. But constantly open extra menu to determine where to go next is rather inconvenient. Therefore, you can press the fast navigation key. A blue beam will shoot from the device on Isaac's hand, which will show you where to go. If there are several targets, the navigator will lead to the nearest one.

Who are the Necromorphs?

Necromorphs are not undead or a form of mutation. It is an alien life form that uses dead bodies as building materials. Necromorphs are unconscious and created solely for killing. After all, the more corpses, the more brothers. Therefore, monsters are unusually active and completely devoid of the instinct of self-preservation. They are just tools controlled by a powerful alien intelligence.

Stasis and kinesis

Sometimes you need to alternate your skills to solve a problem. For example, first slow down the rotor with stasis and drag with kinesis.

The stasis module allows you to temporarily slow down the necromorphs and the mechanisms necessary for the plot. Does the door open and close quickly? Slow her down with stasis and run past. Against monsters, stasis is generally a lifesaver. What is the use to you when the enemy stands still, it seems to me, and so it is clear. You can accurately aim for a shot, run back to a safe distance, dodge the monster from behind, or hold it back until you deal with other opponents. But stasis has a habit of running out, so look for chargers or spare batteries.

Having received the kinesis module, you will be able to move objects at a distance. This skill is usually used to solve simple problems: disassemble a blockage in front of the door, pull a cart to the platform, etc. It is also convenient to collect items with the help of kinesis. Ammunition is on the shelf, but you can't reach it with your hands? A box is lying in the distance, but you really don't want to go to it? So tighten with kinesis.

Kinesis is not a bad weapon at times. The module can not only pull objects, but also launch them at a terrible speed. Therefore, sometimes, especially when necromorphs gather in a crowd, it is a good idea to throw a keg of fuel at the monsters. You can easily recognize fuel containers by their alarming red color. Using kinesis, you can pick up some massive object and block the path of the monster. True, this should be done only for fun, when Isaac is not in danger.

Fighting monsters

Each type of necromorph requires a special approach, so we will analyze the specific tactics of fighting monsters in the bestiary. Now let's talk about general things.

Slow down enemies with stasis so that they do not attack and it will be easier for you to aim.

The first thing to know is the strengths and weaknesses of each weapon type. The Pulse Rifle fires quickly but hits a small area, while the Plasma Cutter deals serious damage, hits a large area, but has difficulty rate of fire. Therefore, the first important point is to choose a combination of weapons that is convenient for you. It is advisable to do this as soon as possible, so that you can immediately start improving the trunks.

The second thing to keep in mind is the way of getting around. Considering that there are a lot of monsters and they suddenly pop out from different corners, and sometimes even fall from the ceiling, walk slowly and carefully. You have nowhere to rush, you will have seen enough of the monsters during the game. Approaching the bend, twist the camera in order to see what is happening there. Sometimes you can see the necromorph before he sees the main character.

Try to use the architecture of the premises to your advantage. For example, you can put fuel barrels in one big pile if you know that a horde of necromorphs will soon run at you. Try to look for secluded corners where the rear and flanks will be covered. If the enemy goes in a crowd along a narrow corridor, this will only benefit you. Necromorphs will interfere with each other, and you can freeze those in front, creating an excellent traffic jam and targets for shooting.

Ancient lure tactics work great in Dead Space as well. But there is one factor that should not be forgotten. Monsters can climb into the ventilation, then suddenly emerge behind their backs. This should be taken into account, but there is no need to be afraid. In addition, if the necromorphs begin to jump out of the ventilation one by one, it will only benefit you. Stand still and shoot the monsters one by one.

The rippers

The Rippers are probably the most common monsters in the game. They were erect, with human arms protruding from the abdomen, and two huge paws-blades growing from the shoulders. At first, they slowly hobble towards the victim, but, coming closer, they make a sharp dash. First of all, shoot their legs to slow down the movement, and then cut off the razor paws to finish off the monster.

Not a very dangerous opponent. The main thing is not to allow being surrounded by these creatures, then problems can begin. There are two types of Rippers - regular and advanced. The latter outwardly differ only in the color of their skin (or whatever they have ...), but much more tenacious. You will have to spend a lot more shots to cut off the limbs. At the same time, the tactics of combat do not change - first the legs, then the arms-blades.

Trade and improvement

You can not only find cartridges and equipment, but also buy. The location of the shops can be viewed on the map. Initially, the assortment wants to keep the best, but as you search for new schemes, you add items to sell. Some items can only be purchased at the store. For example, armor and weapons.

There are five levels of armor, plus a sixth prize, unlocked upon repeated passage. You start the game with level 1 armor. Level 2 armor can be purchased at Level 1 for 10,000 credits. Tier 3 armor will become available at Tier 4 and cost 20,000 credits.

Don't forget to buy new armor as soon as you find a new scheme.

Armor of the fourth level will have to wait until the seventh level, and it will cost 35,000 credits. You will receive level 5 armor at level 10 for 60,000 credits. Finally, Tier 6 armor can be purchased on repeat playthroughs for 99,000 credits.

You can buy cartridges in the store. For this, as, indeed, for any other product, you first need to find a scheme. But I believe that you can safely do without these unnecessary expenses. Ammunition with a reasonable consumption is more than enough, and at the end of the game they often do not even fit into a backpack. If not enough, you need to fight to improve the accuracy of shooting. In addition, prices bite, and if you constantly buy cartridges, you won't have enough money for anything else.

By the way, about the birds. At the beginning of the game, you will experience a shortage of everything, but gradually stocks will grow, and there will be no more space in your backpack. In this case, you can transfer things to the safe through the menu in the store. If the stocks are too abundant, it makes sense to sell some of the items. Do not forget that more often than not you will find ammunition for the weapon that you have on hand. Therefore, be sure to leave in the safe, or better yet, sell the barrels that you don't use.

You can buy first aid kits, oxygen and a stasis battery at the store, but again, I repeat, they can be found at the level without any problems. The stasis battery restores the energy of the stasis module. I do not recommend keeping more than two in your backpack. And the best thing is to do it alone, since there are enough stationary recharges.

There are three types of first aid kits - small, medium and large. The small one restores one health bar, the middle one - two, the big one - all at once. It was enough for me to carry two medium ones with me. Large med packs are relevant only at certain points in the game. Oxygen cylinders restore air reserves and are of three types - small, medium and large. You will usually find them before your spacewalk.

One way to make money is by selling unwanted items. Also, credits are scattered throughout the level, hidden in lockers and from time to time fall out of the monsters. There are special items - microcircuits, which are exclusively for sale. They most often lie in secluded places or hiding places. A gold microcircuit costs 3,000 credits, a ruby ​​one - 10,000 credits, a diamond one - 25,000 credits.

Upgrade your weapons with a workbench.

The last item in the store on the list, but not in importance, is the power block. It is used to upgrade weapons and equipment. But in the game there are rooms that can only be opened by spending a power block. In such a room there are many valuable items, selling which you can usually buy the power unit back and stay in the black. But it's better to record the game first, and then check it out. Each block costs 10,000 credits, and although they are regularly found at levels, if you have free funds, it makes sense to buy an extra one. Upgrades are carried out at the workbench.

Each weapon or equipment has its own scheme of improvements, and often in order to get the desired damage plus three, you have to invest not only in side improvements, for example, increasing the capacity of the clip, but also in empty nodes, which do not give any advantages. Power blocks are always in short supply, so you can upgrade no more than two barrels and some of the equipment to the maximum.

We will talk about improving weapons in a separate chapter. We will only consider the hardware here. You can improve health fourfold by maximum result- plus 100% to the initial indicator. But for this you have to invest in eight empty nodes. Oxygen is also in this diagram. You can extend your stay in space up to 120 seconds. But, to be honest, I don't see much point in investing in this improvement.

At the kinesis module, you can increase the radius to 12 meters. This will require three power blocks. There are no empty nodes in the diagram. A useful but not a critical improvement. Therefore, you can take your time. It is much more useful to invest in the stasis module. You can increase the maximum amount of energy by 75%, and also extend the deceleration time to 30 seconds.

Each parameter will require three power blocks, and you will also have to spend five blocks on empty nodes. The costs, of course, are decent, almost like with health. But the benefits are enormous. You will be able to slow down more enemies, they will remain helpless longer, and you will need to recharge less often, which saves a lot of time.

Where to find items

Finding objects in the game is not as easy as it might seem at first. Due to the lack of normal prompts, it is difficult at first to understand which objects are interactive and which are just interior items.

Obviously, items fall out of the monsters or just lie on the floor. They gleam invitingly from afar, and when you get closer, an image appears with the name of the thing. Items can be picked up by hand or dragged with kinesis.

Also, items are in tall and narrow lockers (like in changing rooms) and low containers. Not all crates and containers can be opened. For closed ones, the red indicator bar is lit, for open ones - a white one. Finally, you can loot ammo and equipment by kicking and kinesis throwing small boxes with a green circular light on the surface.

Weapon

There are seven types of weapons in the game. The first is the Plasma Cutter - you get it for free on the first level. The rest will have to be bought at the store. There is no point in buying everything. You can place four barrels in the quick access slots. Upgrades are enough for a maximum of two, well, maybe three types of weapons. In addition, as practice shows, three cannons are usually used, and the rest are lying around.

Which weapon should you choose? A matter of taste. Each gun has its own strengths and weaknesses, and they all complement each other. Therefore, to say that the plasma cutter is the best, and the rest of the models can be thrown away, would be untrue. I will try to describe the pros and cons of each model based on my own experience. What to choose is up to you. But remember - it is better when theory is supported by practice.

Plasma cutter

The Plasma Cutter is the first weapon you get in the game. It is issued free of charge, while others need to be bought. The cutter fires a fairly wide beam of energy that deals decent damage. True, this is only at the first levels, then you will begin to feel a lack of power, and you will have to invest in improvement. The fully upgraded cutter will remain a valid weapon for the rest of the game.

Alternative fire - barrel rotation. That is, you can shoot both horizontally and vertically. This is very convenient when you need to accurately cut off limbs. They chopped off the legs, unrolled the barrel, and took care of the handles. For example, with a plasma cutter it is a pleasure to gut gut, where special accuracy is required, and you simply cannot find a better weapon against guards or tentacles.

There are seven empty nodes in the upgrade scheme, which is a standard when compared to other weapons, but still quite a lot. First of all, you need to improve the damage, then the capacity of the clip, then the rate of fire and, finally, the reload speed. Monsters drop six rounds, and 25 fits into the clip in the backpack.

Belly

The belly slightly resembles a ripper, only it weighs more and human hands do not grow from the abdomen. Don't be fooled by the size of the monster. He is not at all as clumsy as it might seem at first glance. Seeing Isaac, he quickly rushes forward, swinging huge blades. I don't think you will like meeting him. But the main danger of the monster is in its huge belly. There are dozens of small monsters sitting there. It is worth shooting in the stomach, as they will instantly be free.

When you see a belly, try to slow it down with stasis. Then cut off the legs with shots, and when he starts to fall, carefully shoot off his head. As a result, the monster is dead, but the belly is intact. If, nevertheless, the monsters were released, switch to a flamethrower or some other weapon that hits the area, and burn the creatures. If one or two bastards jump on Isaac, nothing bad will happen. Press the indicated key quickly to crush your opponent. But if the whole swarm attacks, the hero will be devoured alive.

Laser rifle

The laser rifle has such a wide range of damage that there is no need to accurately aim to dismember monsters.

The laser rifle works like a plasma cutter - it fires a wide beam of energy. Only this weapon has a larger area of ​​impact, a stronger shot, plus you cannot rotate the barrel. Let's talk about the cons right away. The first is a low rate of fire. It is much smaller than even a plasma cutter. The second is a small clip, both at the weapon itself and in the backpack.

The pluses, first of all, include the monstrous power and the area of ​​destruction. Best weapon if you want to make minced monsters. My favorite trick is to slow down a few monsters, wait until a crowd gathers in a narrow corridor, and slash at the legs and arms. If damage upgrades are made, only a few will survive. True, with the belly, this automatically guarantees additional problems. With this approach, aimed shooting does not smell, so have a weapon against the swarm at hand.

An alternative firing mode is a mine. Before jerking, it spins for a while, and then shoots deadly rays in all directions. Not a bad crowd attack, but you want the monsters not to run away, so use stasis. If you choose this weapon, prepare to give up most of your backpack for ammo. Too little fits in one clip.

There are nine empty nodes in the circuit, which is not great at all. First of all, you need to invest in damage. Then, if there are problems with the space in the backpack, start improving the capacity of the clip. If there are no problems, then first in the mine, and then in the cage. Then into the angle of attack. Finally, the last thing to improve is the reload speed. Monsters drop two rounds, a clip in a backpack holds six.

Pulse Rifle

The Pulse Rifle is not very powerful, but it is convenient to shoot at moving targets with it.

The Pulse Rifle is a controversial weapon. On the one hand, there is little damage, and it is inconvenient to cut off limbs with it. But thanks to the high rate of fire, if you count the damage done per second of time, it turns out well. True, the higher the level of difficulty, the less this plus. Monsters become tenacious. In addition, a decent amount of cartridges fits into the clip, which greatly saves space in the backpack.

I usually use the Pulse Rifle against small and nimble creatures. If you miss, you don't have to waste time recharging. And if you hit, the necromorph will stop and you can finish him off. Also, it turned out to be convenient to destroy level bosses from a rifle. You can almost instantly switch from running to shooting and not worry if some of the charges miss the target.

An alternative attack is a circular shot. Isaac raises his weapon up and fires 360 degrees. Not very useful because it wastes a lot of ammo and suffers from range with accuracy. But if the enemy is surrounded, you can use it for defense. Having used up the clip, you will surely finish off someone, thereby creating an escape corridor.

There are seven empty nodes in the diagram. What should be improved first? At first, I would double the damage so that the monsters live worse, and then I would invest as much as possible in the capacity of the clip. With 175 rounds in the rifle, you will not only free up your backpack, but also increase your efficiency in combat. You can leave the reload speed for last or not make this improvement at all. 25 rounds fall from monsters, a hundred fits into a clip in a backpack.

The guards usually hang in front of the doors and prevent the passage. They look like a shapeless pile of meat with a human head and four tentacles protruding from their chest. These monsters cannot walk, but that doesn't make them harmless. If you get closer to them, they will blow off Isaac's head with one blow of their tentacles.

When the guards spot the character, they begin to spit out pieces of meat from the belly. A second after the appearance of the pod, a tentacle grows out of it, from which it begins to shoot. You can kill the pods by cutting off the tentacle, but until you finish off the parent, they will constantly spawn. The Guardian can be killed by cutting off all four tentacles. This can take a while because you have to aim at a distance. Therefore, it is advisable to slow it down with stasis so that the monster does not spawn pods.

Flamethrower

The industrial gas burner is perhaps the most unfortunate weapon in the game. It is convenient for them to burn small creatures, but, perhaps, that's all. Healthy monsters can be fried for a long time and persistently, and they will not experience much discomfort. Often it is not enough for one necromorph and a whole clip. Plus, the flamethrower doesn't work in space. After improvements, things are better with damage, but useful recoil comes too late.

An alternative attack is a shot with a ball of fire, which explodes after a while. Here it does more damage, but is rather difficult to use. If you nevertheless decide to seriously use a flamethrower, first of all, improve the damage. Then the duration of burning, then the capacity of the cylinder and the recharge rate.

There are eight empty nodes in the diagram, which, in my opinion, is too many. 25, ahem, cartridges fall from the monster, and in total 150 units fit into the balloon in the backpack. This is, however, quite a bit at the rate of fuel consumption.

Power pistol

The power pistol not only deals damage, but also knocks back the monster.

The power pistol uses the same technologies as the kinesis module. A powerful force blow is inflicted on the enemy, which breaks bones. If the plasma cutter is a scalpel, then the power pistol is a huge club. A normal shot not only cripples monsters, but also knocks them back. Therefore, even if you do not finish off the necromorph, at least throw it away from you.

An alternative attack is a ball of energy that penetrates and then scatters opponents. In general, a kind of grenade. The power pistol is not an accurate or long-range weapon, it can even be compared to a shotgun. The weapon works best against small enemies, as well as in chapters 4 and 9, when a monster can be pushed into a zone of disturbed gravity and thereby instantly finish off without spending extra cartridges.

There are seven empty nodes in the diagram. First, as usual, raise the damage, then the speed, then the reload (it is too slow) and the capacity of the clip. Enemies drop three cartridges, 15 fits into the clip.

Pipe cutter

The main source of ammo in Dead Space is gutted monsters.

Having suffered with a flamethrower, I bought a pipe cutter and realized that this is an excellent replacement with much more functionality. The pipe cutter is also a melee weapon, so it is advisable, but not always necessary, to slow down the enemy with stasis. Then press the fire key. The pipe cutter spits out a circular blade that rotates a specified distance from the tool. Until the blade stops, you can control its flight.

If the monster is inhibited by stasis, slowly cut off its limbs and watch how it nicely crumbles into pieces on the floor. If the necromorph is active and hungry, quickly drive the saw over his legs to first immobilize, and then saw off the remaining limbs. Small monsters are destroyed in the same way. Freeze them until they crawl to the corners, and crawl with the saw until the necromorphics stop twitching.

An alternative attack is a saw shot. It pierces all enemies in its path, but a small area of ​​destruction does not give a tangible result. No matter how much I tried to adapt this attack to different situations, nothing sensible came out. The low rate of fire negated all the advantages obtained.

There are eight empty nodes in the diagram. Of course, we start with the damage, then we will certainly invest in the length of the sawing. In principle, this is enough, but it's still a good idea to do reload and capacity (in that order). Monsters drop 4 discs, 15 can fit into a clip in a backpack.

Power cutter

Always keep an eye on the number of cartridges so as not to start reloading at such an unfortunate moment.

The Power Cutter, like the Power Pistol, is an area-impact energy weapon. Only he has a different principle of work. First, you hold down the shot key and accumulate energy. When it reaches its maximum, depress the key to start the charge. One shot is usually enough to destroy several Necromorphs.

The catch, in fact, is that you need to accumulate a charge. This is not always the case. Enemies do not like to stand and wait for you to deign to shoot at them. And using it not against a crowd of monsters is a waste of ammunition. Not only is the clip of the cutter small, but only four cartridges fit into the clip in the backpack. There is a catastrophic lack of space, and after a serious mess there will be a shortage of ammunition.

Alternate Shot - Isaac points the barrel down and fires. The attack knocks the enemy down while dealing damage. Works well against small monsters and when you are surrounded.

Let's summarize. Yes, the Power Cutter is a lethal weapon, but using it against single monsters is like shooting a sparrow with a cannon. The weapon is effective against the crowd. Even so, I preferred to use the laser rifle, which does not require pre-charging before firing.

There are twelve empty nodes in the scheme - this is an absolute record among all weapons. The first thing to invest in is damage. Then go to charge, smoothly switch to clip capacity and finish with reload speed. One cartridge drops out from the enemy, four can fit into the clip in the backpack.

Passing

The small repair ship "Kellion" is slowly approaching the gigantic "Ishimura", which suddenly stopped communicating. This is not to say that this is a typical assignment. It is surprising that on a ship of this class and scale, all means of communication were out of order at once. Extinguished side lights are even more worrying.

Docking starts smoothly, but suddenly the Kellion shuttle is dramatically pulled by the gravitational field, which is why the landing takes place in an extremely hard mode. The crew survived, but the ship was seriously damaged, therefore, in order to get out of the Ishimura, the engines will have to be repaired. But while there is no question of flight, first you need to contact the Ishimura crew, who for some reason are in no hurry to get to the landing site.

However, let me introduce you to actors... Isaac Clarke is an engineer and repairman, as well as the protagonist of the game. Zach Hammond is a security officer and Kendra Daniels is a computer systems specialist. We will not introduce two pilots, they are destined for the role of the first victims ...

Arrival

After landing, wait until you return control, and go to the room at the stern to pick up the first aid kit. Get out of the broken shuttle and go to your comrades. Open the door and enter the waiting room. The elevator is not working and needs to be energized. Go to the room on the left, save the game and use the computer.

The first date with the monsters we will have to go through with bare hands. We'll have to get away with all the strength.

The elevator started working, but monsters appeared and quickly devoured the pilots. The beauty of the situation is that we have no weapons, so all that remains is to run. It is impossible to get lost, the main thing is not to slow down. Once you reach the elevator, quickly press the button to get away. When the lift stops, search the room and pick up the plasma cutter from the table.

Approach the closed door and destroy the panel with a shot. The passage is now clear. Finish off the monster, collect all the items and go to the transport system control room. Through the glass you will see Hammond and Kendra, who managed to escape. First of all, you need to establish a transport system so that it is possible to move around the ship. Without this, it will not be possible to get out of the Ishimura.

Exit the control room. Go forward, carefully collecting all the items. At the beginning of the game, literally everything will come in handy. When you see the corpse of a monster, shoot him in the legs to stop pretending. After reaching the large room, pick up the stasis module - now you can slow down enemies and mechanisms. Go to the broken door, shoot it with stasis and run into the corridor.

Keep moving forward collecting all the items until you reach the repair shop. Take the power block from the box on the wall and activate the left grappling mechanism. It will hook onto the transport cart. Finish the appeared monsters and go to the second remote control. The gripping mechanism is knocking, therefore, using the remote control, you need to immediately shoot at the "claw" with stasis.

When both grippers are hooked on the transport cart, use the center console. The broken cart has now been removed from the tracks, but the repair is not yet complete. Therefore, deal with the monsters and go to the jammed door. Look on the map where you need to move next. The previously locked door is now open. Now we have to fight through to the key, which is needed to open the storage room.

Despite the creepy appearance, human features are clearly guessed in the monsters. Which makes it even more creepy.

Having obtained the key, go to the storage room. I'm not talking about monsters - they will delight you all the way. In the storage room, take the power block and upgrade your weapon or equipment on the layout. Take the data and leave the room. You need to go back to the control room.

You know the way, it remains only to break through the necromorphs. Download the data to the computer and use the second remote control to activate the transport system. Hammond and Kendra will go to the bridge, and you need to get the Kellion ready for flight. Look at the map and hit the road. When you get to the waiting room, you will be contacted by Hammond, who made it safely to the bridge. There is no one there, but the computers are still working.

Well, it's just great, what else to say. Go to the "Kellion", go into the cockpit and use the computer. Something clearly went wrong as the ship began to explode rapidly. Run out of the ship and immediately aim at the exit from the waiting room. From there, a crowd of monsters will tumble down, which you need to quickly shoot.

After the battle, Kendra and Hammond will contact you. Now you have no shuttle, which means that your chances of getting out of the Ishimura are diminishing. However, let's not despair. As long as we are alive, there is hope. We need to find the captain's body to get the access codes. To do this, you need to get into the medical compartment. Hammond has already sent the transport cart back, so quickly run to the station to finish the level.

Small and nimble creatures that crawl along walls and ceilings. Seeing the hero, they take a comfortable position, raise three tentacles and shoot. The charges fly in a wide arc, making it difficult to dodge the shot. Crawlers are easiest to kill when they are preparing to attack. Wait until they raise their limbs and shoot them carefully. To prevent the monsters from attacking, you can slow them down with stasis.

You can try to shoot the crawlers when they are moving, but due to the high speed this is not so easy. Stasis doesn't help much either, because the energy charge travels even slower than the shot. There are two types of sliders - regular and advanced. The latter differ in skin color and the amount of health.

Intensive therapy

When the trolley stops, go to the wounded and dying doctor, who will hand you the kinesis module. Use kinesis to clear the trash at the door and enter the hall. Pick up a flamethrower schematic and upload it to the store. You can now buy an industrial burner. Hammond will also contact you. He survived the monster attack but lost Kendra. However, this does not change our plans - if we want to get off the Ishimura, we need to take the access codes from the captain's body.

There are no absolutely safe places in the game. If you don't ransack the room, you may be attacked even when you are shopping.

The door to the morgue is littered with debris, and in order to remove the blockage, we need special equipment. Hammond reveals that he has found a thermal mixture that can burn a hole. So make the purchases you want, look at the map and hit the road. Slow down the door with stasis and enter the lab. Clear the room from monsters, collect items and go down to the lower level. The security system will detect the presence of monsters and close the doors. This is your signal, so keep your weapon ready.

Before entering Kane's office, use kinesis to move the bookcase to find the cache. Then collect all the items, record the game and go to the biolaboratory. You will see a scientist who, like all the Ishimura's crew members, will be quickly gnawed by a necromorph. Meet the new monster - the crawler. Wait until the scientist throws away the skates, go into the room and shoot the creature.

Collect items and use the elevator to get to the second level. Here you will find a small surprise, after the death of which you will be able to pick up a thermal mixture and other items, including a power block. Return to the main room and use stasis to slow down the door in order to slip through without fear of being chopped into two parts.

Go to the rubble ... No, you can't use the thermal mixture yet, so after shopping in the store, look at the map again to figure out where to go now. Once in the hall, raise your weapon to illuminate the road with a flashlight. Ahead of you is a wonderful sight - a corpse beating its head against the wall. You can relax, now nothing will happen. This is solely for pumping the atmosphere.

In the next room, you need to use kinesis to move the platform to get to the other side of the room. Fight off the running monsters and collect all the items. Then search the rooms. There is also a lot of interesting things lying around. Then return to the main room, use kinesis to pick up the power supply and insert it into the receiver in the wall to supply power to the lift.

Pull up the platform to get to the other side and finish off the monster. After going around the wall, drag the platform again to get to the door. Use a shot to knock out the control panel to open the door and go inside. In the next room, a meteorite made a hole, and you will find yourself in open space. Therefore, keep an eye on the oxygen levels so that Isaac does not die of suffocation. But there is no need to hurry too much, there will be enough time and air supply. So take your time to search all the lockers.

Swarm monsters are dangerous when attacked in a crowd. The character is gnawed instantly.

There is no gravity in the next room, so get used to the idea that you can walk on the ceiling and walls, as well as jump tens of meters forward. Fly to the door, grab the power supply floating nearby and insert it into the receiver in the wall. Take the item we need, collect the money with the ammo and go back to the zero gravity zone. There are already two monsters swimming there, so learn to fight in new conditions. Now such fights will occur quite often.

Go out into space, finish off the necromorph and return to the morgue. You now have the items you need and can burn through the barricade. Go back the old way, only this time small monsters will be waiting for you. Slow them down with stasis, and then burn them with a flamethrower. After reaching the barricade, plant the bomb and wait until it clears the way. So we got to the medical compartment.

Clear the room of monsters, collect items and insert the power supply into the receiver to open the door. Now it remains to fight through to the chamber intensive care... Ignore the crazy doctor dealing with the sick, but move on immediately. Several unpleasant surprises await you along the way, but in the end you will take the elevator to the morgue. You will see how the captain is turned into a necromorph. Well, let's wait until we can do anything ...

When the Necromorph Captain and Infector rush at Isaac with wild screams, shoot the monsters and collect items. Be sure to take the access codes and the power unit. You just have to return to the transport station. Along the way, you will be sluggishly disturbed by monsters. While you stomp along the corridor, Hammond will contact you and say that the monsters are not the result of mutations, but an alien life form. But this is nonsense. He is much more concerned about failed engines. If we don't fix them, the ship will simply crash to the planet.

Infectors

These necromorphs look like flying rays. Their primary task is to create new monsters from corpses. Therefore, they usually do not pay attention to Isaac, preferring to create help for their fellows. Now, if there is already no one to convert, then they will definitely attack.

Infectors need to be killed first, before they have time to infect the corpses. Miss this, and you will soon be assured of a meeting with new monsters. These monsters are weak enough and die from being shot in the body. But for the best result, try to shoot at the wings.

Orbital correction

Your global tasks at the level are to start the centrifuge and refuel the engines. Collect all the items and an audio recording from which you will learn about the events that took place on the ship. From the save point, go right and shoot at the supposedly dead monster. Don't pretend to be a bastard.

Kill the infestations first, before they make new monsters out of the corpses.

Move on, remembering to collect all the items while visiting the shop and workbench at the same time. Clear the corridor from the monsters, and then use kinesis to lower the switch to activate the power. Go to the cart and pull it up with kinesis. Sit inside and use the remote to move to the other side. The delegation is already meeting you there, so get your weapons ready. After collecting trophies, find the power block and climb the stairs to the second tier. Select the pipe cutter circuit and use kinesis to activate the second remote control.

Shoot the monsters that have appeared, return to the cart and drive back. Now go to the control room. Through the glass you will see how the infestator is courting the corpses, so get ready to meet the necromorphs. Take the elevator to the next tier. Search the lockers, upgrade the weapon if you wish, and go to the disinfection room. Use the remote control to start the decontamination process.

True, things will not go according to plan. The light in the room will go out, and monsters will crawl through the ventilation. Raise your weapon to turn on the flashlight and fire back at the necromorphs. After repelling the attack, you will unblock the doors and be able to go outside. Through the corridor - there are a gold microcircuit and a power unit - you will find yourself in a room with a centrifuge. Finish off the monsters so they don't interfere with the task.

The room has zero gravity, so feel free to jump to the lower tier. You need to connect two large rotors to the centrifuge. But in order for them to successfully join, you must first slow them down with stasis. When both rotors are connected, go up to the second level and use the remote control.

Gravity returned to normal, but the air evaporated. You will have to quickly get out of the room before the character suffocates from the lack of oxygen. Take the elevator down to the first tier, and when the rotor rushes past you, quickly run to the next cover. Do not forget about the monsters. In one of the shelters, you can recharge your oxygen supply. After reaching the elevator, go upstairs and leave the room. But this is not the end of the adventure. When you find yourself in the corridor, Isaac will grab a huge tentacle and drag him down the hole. Shoot the yellow bag to destroy the giant limb.

To free yourself from the grab, shoot the yellow bag.

Take a breath, look on the map where to go, and run to the elevator. You will be contacted by Kendra, who will start pumping Isaac. useful information... In the control room, visit the store, save the game and go to the fuel storage. You will have to go through the destroyed part of the ship in open space. There is no direct path, so you have to jump over the rubble. Just do not forget to collect items while doing this.

Once in the whole room, collect items, among which there is a power block, and take the elevator to the next tier. Continue to move towards the goal, trampling on small monsters that you meet. Go to the broken door. Wait until she chops off the necromorph, slow her down with stasis and slip inside. Shake a can of fuel on small creatures, reload all weapons and go into the storage with fuel.

There are a lot of monsters inside, but there are also enough fuel cans, so half of the necromorphs can be killed without spending ammo. After destroying monsters, reload your weapon, restore health and use the remote control. Now you need to reflect the second attack. Try to gather enemies in a heap so that you can cover everyone with a laser rifle or throw a can of fuel if there are any. The battle will be difficult, so do not relax and, most importantly, do not stand still.

When you finish off all the monsters, you will be released from the room. Go to the transport cart. On the way, you will meet only one necromorph. Hammond will contact Isaac and say that they are no longer in danger of falling to the planet. But then Kendra will call and inform that the protection from asteroids is not working. This means that if nothing is done, the ship will soon be destroyed by boulders.

Tentacle

Sometimes the main character will grab a huge tentacle and drag him down the hole. It cannot be slowed down, but it can be destroyed. When the tentacle stops to rest, shoot the yellow pouch. You always have enough time to hit the monster.

Death is inevitable

It looks like Kendra is not more concerned with the meteorite attack, but that Hammond knows about the obelisk. While the comrades are scolding, collect the items and go to the wheelhouse. You can ignore the appeared and disappeared paw. We will get acquainted with its owner a little later. On the bridge, you will witness a meteorite hitting the ship. Everything around will rumble and begin to crumble. Calm down - this is not a reason to panic, without a script, nothing will collapse to the end. Therefore, thoroughly search the boxes, save the game and go downstairs.

Freeze the creature with stasis and shoot in the back - there is simply no other vulnerable spot.

Find a level 3 costume scheme. Now you need to take it to the store as soon as possible in order to immediately buy new armor for 35,000 credits. In the meantime, go to Hammond's control room. He will say that he knows nothing about the obelisk, and Kendra is extremely impressionable. And in any case, if Isaac does not fix the asteroid defense system, this will no longer be relevant.

Hammond can activate the system, but for that you need to redirect the energy. And first you have to start the elevator. Go back to the bridge. When you come to the elevator, the door will be broken down by the creature. Her armor can only be pierced from the back, so first brake with stasis, go around from behind and shoot until the monster dies. If you have upgraded your weapon, it won't take long.

Pick up the trophy and go to the room where the monster jumped out. Search all the lockers, activate the power and go to the elevator. Be sure to watch the recording while descending to the lower tier. Pick up the Power Cutter Circuit and use your shot to wake the sleeping monster. Then, with a clear conscience, step towards the server room. Kendra will contact you and say that there is some gravity problem in the room. If you step on a piece of floor with a distorted field, Isaac will be torn to pieces. This also applies to monsters.

Carefully bypassing the broken plates, go forward, knocking out the monsters. If you have a power pistol, you can throw their monsters into fields with disturbed gravity. True, then trophies will have to be selected carefully. Reload the stasis and enter the server room. There you will be attacked by another creature. Slow down the necromorph with stasis and shoot him in the back while he helplessly fiddles with his paws.

Pick up the power unit from the corpse and go to the corner of the room. Slow down the electrical cable when it is on the left side, quickly run up to the remote and turn off the power. This will redirect the energy in the first circuit. Return to the wheelhouse, finish off the necromorphs and go to the elevator. You need to go down to the lowest level. While you are riding the lift, Kendra will contact you and tell you that necromorphs appear by transforming dead tissue. Some monsters make corpses, others create new creatures.

Upon arrival, take a look at the map to see where to go next. As soon as you go forward, the partition collapses and you find yourself in open space. Run faster to the next room. Deal with the monsters and, bypassing the violated zones of gravity, go to the control panel. Redirect the energy in the second circuit and return to the elevator. Destroy the oncoming necromorphs, go into the lift and go to the third level.

A blow of your hand can temporarily stun a monster in order to run away or finish off a necromorph.

Exit the elevator and collect your things. Among other junk you will find a diagram of a power pistol. Prepare your weapon and slowly move forward, getting ready to repel a massive attack of monsters. Once the path has been cleared, use the remote to redirect the energy in the third circuit. Hammond will contact you and inform you that the defense system is not working yet, but the cannon is activated and can be controlled manually. But to get to the gun, you have to walk around the hull.

Drive to the next level, collect oxygen tanks and head outside. The ship is periodically bombed by asteroids, so run during periods of calm, and hide behind steel shelters the rest of the time. After reaching the gun booth, pick up the ruby ​​microcircuit and the power unit and sit in the chair. Your task: to destroy the asteroids until Hammond activates the defense. At the beginning of the mission, the protection of the hull is 85%, if it drops to zero, Isaac will die. In general, the usual shooting range. Just don't overheat the cannon, shoot in short and precise bursts.

When Hammond turns on protection, you will complete a global quest at the level. The ship is safe, but now something is poisoning the air. You urgently need to eliminate the threat, otherwise you will simply suffocate from the poisonous fumes. Hammond goes to the hydroponics section, Kendra struggles to solve the problem. It's time for you to go too. Run to the transport cart. There will be monsters along the way, but this is nonsense.

This necromorph is certainly dangerous, but with the right approach, you will defeat him within one to two minutes. You will always have to fight the creature one on one. The creature loves to rush, taking out thin walls and trying to crush Isaac. From afar, he can launch explosives, which, however, can be caught with kinesis and sent back to the owner.

The creature is covered with impenetrable armor in front. You can cut off the legs as much as possible, but the necromorph won't lose much in speed. Therefore, wait until the monster rushes at you, slow down with stasis and run from behind. This is his weak point. Shoot in the back with the most powerful weapon, remembering to immobilize in time with stasis and bypassing the monster when it starts spinning. After defeating the creature, you will receive a power node.

Deadly haven

After exiting the cart, wait for Kendra's message. The poison is spreading faster and faster in the air, and you need to hurry to the laboratory to get the antidote ingredient. Go to the security room and you will be contacted by some religious fanatic who will start broadcasting about natural selection and divine design. Collect items and stomp towards the diagnostic room.

Do not forget to improve your weapons, because the monsters are gradually becoming healthier and healthier.

Finish off the two crawlers and head to the elevator. Shoot the guard on the wall and take the elevator to the second floor. Slow down the rolling platform and jump to the other side. If you do not have time to slip, Isaac will be fried with electricity. Finish off the next crawler, go around the wall, slow down the platform again and run to the door. Take the elevator down to the laboratory and enter the room on the right. Take the circuit and the power unit and activate the remote control to receive the reagent.

When you pick up the reagent, the flaps on the window will open, and you can chat with the crazy doctor. The doctor will again begin to carry religious nonsense about the fall of humanity and the necromorphs as a new and ideal form of life. In general, everything is clear with him. But the doctor will decide not to limit himself to words and set his brainchild - a hunter on Isaac. This creature is almost immortal, so chop off its arms and legs, throw stasis into the stump to slow down the regeneration, and run out of the room.

If Kendra didn't manage to unlock the lock, she'll have to wait a bit. Shred limbs and constantly freeze with stasis until you can leave the room. Run to the diagnostic room. There are monsters crawling. Stop them with stasis and burn them with a flamethrower or shred them with a pipe cutter. If you feel sorry for the cartridges, you can throw the fuel tank, which is in the corner.

Once you're safe, take a break and listen to Kendra. Now you need to get to the clinic to get your DNA samples. Look at the map to see where you need to go next. In a small room, you will once again talk to the doctor through the glass. Then you will go out into a large room. Here you will be met by a hunter with other necromorphs. Dismember and freeze the hunter immediately, and then take on the rest of the monsters.

When you kill regular necromorphs, the doors will automatically open. But if you want to collect things, do not forget to slow down the hunter in time. Once in the next corridor, you can forget about the immortal monster for a while. Collect items, lift the canister and head down the hallway. Bang the guard to the left of the door with a balloon and search the corridor to the right. At the end of it there is a maddened nurse who only gets on her nerves with insane laughter. Pick up items and move on.

Go to the demented doctor's office, collect items and use the computer. When the DNA samples are ready, take the capsule and quickly run out into the hall. Our madman let out the air, and now we need to quickly get to safety while there is something to breathe. In the corridor, slow down small monsters with stasis and run past.

All the members of the ship "Ishimura" are already dead or, like this nurse, a little out of shape.

In the large room where you met a hunter and a bunch of monsters, you will again meet a hunter and a bunch of monsters. Fighting them is not at all handy, so the best option would be to cut off the hunter's legs, slow down the other necromorphs and quickly run past. True, if there is no spare cylinder, you will have to replenish the oxygen supply from the apparatus on the wall. This may delay the flight somewhat.

In any case, in the security room, use the remote control to seal the compartment. Now you need to make poison, so it's time to visit the chemical laboratory. As you stomp to the target, Kendra will report that, most likely, the air is being poisoned by a huge Leviathan creature that has settled in the hydroponics compartment. Once you reach your goal, create poison, recharge your weapon and stasis module, and hopefully step into the cryogenics laboratory.

There you will meet the doctor again, who will reveal his plans. He wants to take the necromorphs with him to Earth in order to bring all the inhabitants of the planet to divine happiness. Oh, how smart! True, we cannot get to the doctor yet, but now we will have to fight with the hunter. No escape this time.

Traditionally, dismember and slow down the hunter first. Then quickly destroy the crawlers. Run into the control room where the doctor stood and wait for the hunter. Dismember and immobilize again. Return to the main room and wait for the monster. Lure him into the freezer section right in the middle of the room. When he enters the chamber, chop off the limbs again and brake with stasis. Run to the control room and use the remote control to freeze the hunter forever.

Now there is no problem. At least for a while. Take the power block from the box on the wall and run into the transport compartment. You are all set to poison the Leviathan and stop air pollution. Time to go to the hydroponics bay, where poor Hammond must have turned green from the necromorph filth.

These necromorphs, with whom we will have to fight twice, are the kind of bosses of the levels. They cannot be killed with conventional weapons. Even if you cut off all the limbs, the monster quickly regenerates and rushes after you again. You have to kill with improvised means. Until you reach them, and this will happen only as you move along the level, you will only have to flee. What's the best way to neutralize a hunter? Cut off his legs, then his arms, slow down with stasis, and start running away.

Dangerous impurities

Collect all the items at the station and go to the elevator. Go down to the lower tier and find Hammond. He is still alive, but already barely breathing, so we need to hurry up and clear the air faster. But a friend can still help with advice. He learned that killed and converted employees in the hydroponics section were spoiling the air. They need to be destroyed before coming to grips with Leviathan.

Purchase the items you need from the store and head to the west wing. You will be contacted by Kendra, who will once again express her distrust of Hammond. Well, of course, he's a security officer, who else to suspect? Go to the door on the left. Hot steam occasionally escapes from the pipes in the corridor. In order not to burn yourself, wait for a pause between emissions and quickly run through the dangerous area.

It is convenient to shoot monsters in doors and narrow corridors. They have no wiggle room.

Move forward carefully, shooting monsters. When you reach the large hall, reload your weapon, become Mister Caution and quietly move forward. Snarling necromorphs will periodically jump at you from different sides and try to tear you to shreds. After defeating them all, search the hall and alternately visit the small rooms on the right and left sides. There you will find one poisoner.

Of course, when you finish off the poisoner and go out into the hall, you will traditionally be greeted by necromorphs. So, two creatures are ready, and you can move on. Take the elevator to the third floor. Pick up the items, go down to the second level and step into the room behind the elevator. Nicole's image will tell you that you are on the right track. Shoot the poisoner, quickly turn around and throw stasis at the infestations before they turn the corpses.

Return to the elevator and step into the room opposite. In a long corridor two demolition men will attack you. Shoot the orange bag on your left hand before they get too close. You will then exit into a zero gravity room. Jump to the hatch and use kinesis to open the lock. Finish off the monster and, carefully, so as not to fall on the spikes, jump further. You will have to suffer a little, but in the end you will achieve the goal.

Go to the elevator and get down to the lower tier. Now you need to go through two corridors, which are periodically burnt out by jets of flame. Without stepping into the first corridor, shoot the control panels to unlock the doors. When the flame subsides, run to the middle corridor - there is a safe place here. Fight off the monsters, wait for the flame to subside in the second corridor, and run through it. It remains to reach and finish off the poisoner. Now return to the great hall, and from there step into the room from where you started your journey along the west wing. Sell ​​the extra items in the shop and head to the east wing.

A guard will greet you here. Cut off his tentacles before he spawns monsters. Search all the rooms to collect many useful items and go to the large hall. There, a creature will attack Isaac. You already know how to deal with it. Brake with stasis, run from behind and shoot in the back from the most powerful weapon. Pick up the power block and stomp to the poisoner. Nobody will try to stop you.

After finishing the living barrel of chemicals, go to the control panel of the power system and press the button to raise the panel. When it opens, stop it with stasis and look inside. Oh, another poisoner hid. Finish off the infection and use the elevator in the center of the hall to go up to the second floor. Collect everything that is bad, go into the room and again collect everything that is bad.

Yellow spots are vulnerable spots. This rule applies not only to tentacles, but also to other monsters.

Next, Isaac will grab the tentacle and try to drag him into the hole. Shoot the orange knot to place it on the plans. Having freed yourself from the strong embrace of the monster, confidently step into the zone of zero gravity. Do you see a faulty electrical appliance that shocks the floor? If you step on the panel, you will instantly die. Therefore, wait until the unit stops sparking, throw stasis into it and quickly jump over the danger zone. Finish off the Necromorphs. Do not forget to turn off the electricity, the remote control is next to the device. Now go to the poisoner, shoot him and the old road return to the corridor where Isaac was attacked by the tentacle.

Go to the elevator and go to the third floor. The last poisoner is sitting there. You will reach it without incident, therefore, having eliminated the last source of toxins, return to the large hall and fight your way into the central room with battles. Use the computer mixture obtained on the fifth level to poison the Leviathan. The poison will affect the monster, but will not kill. Well, you have to figure it out the old fashioned way, with the help of guns. The battle will take place in a zone of zero gravity, so make the appropriate conclusions.

Despite its impressive size, Leviathan is not a very serious opponent. First, he will thresh with tentacles. Run from side to side to dodge an attack. When the tentacle hits the ground, you have a few seconds to fire at the orange knot on the monster's limb. Thus, you must destroy three tentacles. The battle will move into the second stage.

Leviathan will continue to attack with tentacles, and much more actively than the first time. But now we are not interested in them. You need to explode yellow balls in the mouth, which play the role of projectiles. If you hesitate, the monster will launch them at the main character. You can grab them with kinesis and throw them back, but it's easier to just jump to the side. By detonating a certain number of balls in the mouth, you will finish off the Leviathan. Which means it's time to head to the transport bay. Moreover, Kendra has figured out how to send an SOS signal.

Demoman

The Demoman is a terrorist among the Necromorphs. His only goal is to limp to the main character and blow himself up. The explosion not only removes a lot of health, but also knocks the character down. The Demoman is not a very harmful monster due to its low movement speed. By and large, it is only dangerous in confined spaces.

Usually I was able to use the Demoman against other monsters. Stasis slow down the monster and wait until a larger crowd forms next to it. You make one shot at the yellow bag on your left hand, and almost all opponents die at once. Beauty! The savings in ammunition are capital.

Step into the void

Kendra's plan is simple enough in theory. You need to install a beacon on an asteroid and launch a rock into space. With a high degree of probability, he will fly far, far away, and someone hears a call for help. After leaving the transport cart, collect all the items in the surrounding rooms - there are really a lot of them - and take the elevator down to the processing room. When the lift starts moving, monsters will start jumping down from above. Therefore, stand in the corner and shoot from the cannon more powerful.

Cut off the legs of monsters to slow down your opponent. A creeping reptile is much easier to finish off.

First, there is a rather nervous walk along the corridors, surrounded by necromorphs. Then, through the gap in the wall on the left, you will reach the ore processing hangar. You need to restore the force of gravity in the room in order to go further. But for this, you first need to remove all dangerous and unstable elements. Grab small asteroids flying around the room and throw them into the power beam. Twice - after each successful throw - you will be disturbed by monsters, so do not lose caution.

When all the asteroids are destroyed, go to the remote control in the corner, but do not rush to press the button. As soon as you restore gravity, huge crowds of monsters will run from both sides. Therefore, it makes sense, at the cost of a power unit, to hack into the room at the remote control and keep the defense there. Necromorphs will jump from the ceiling, but one at a time, and you can easily interrupt them. And the prizes found in the room will allow you not only to buy a new power unit, but even to stay in a small plus.

Then go to the unlocked door. Pick up the access key, collect other items and return to the elevator. Just don't forget to sell any junk in the store. You won't get to the outlet soon. Then go to the elevator and go to the lowest level. This time, no one will try to devour you.

After exiting the elevator, quickly finish off the infestations while they spawn monsters. Then use kinesis to pull up the cart and ride it to the other side. On the right and left, monsters will jump out and shoot at Isaac with all sorts of nasty things. If you quickly destroy the necromorphs, you will even have time to collect all sorts of prizes scattered on the side platforms. When you reach the point, you will see Nicole on the other side. It turns out that this is still not a hallucination? She will open the door for you, but you need to cover the girl.

The Necromorphs will mostly try to reach Nicole, but will attack Isaac somewhat. In any case, you should think about the girl first. A few strokes are enough for her to go to heaven. After completing the task, you will gain access to the room, but Nicole will escape. Well, we can't change anything. Go into the room, pick up the lighthouse, the scheme of the fourth level suit and other useful things. Then return back on the cart. Monsters will attack you again, so keep your weapon ready. Make your way to the elevator - the contaminants are back in business - and ride to the second tier.

Sometimes monsters literally jump on Isaac. To remove the enemy, quickly press the key indicated on the screen.

The room where you will find yourself upon arrival is completely invisible and full of monsters. Therefore, lure them to the elevator, where it is much better with visibility and easier to aim. Go to the room to the left - there is a store, you can buy armor, - remove the power supply from the receiver, throw it, return to the dusty room and finish off the monsters that have appeared. Pick up the power supply again and carry it to the room opposite. Insert the unit into the receiver to activate the lift. You can now reach the asteroid.

Having dealt with the monsters, carefully examine the hangar. You need to destroy the four rotors that are holding the stone. The first is on the floor, the second on the ceiling. Where exactly, you can understand by the force beam coming from the asteroid. You are in a zero gravity zone, so getting to the rotors is not a problem. Slow down the mechanism with stasis and shoot at the power supply through the hole in the side protection.

The remaining two rotors are on the other side of the asteroid, and you will have to get out onto the hull of the ship. Wait for the clamps to release, jump onto the rock and quickly run out. Shoot the necromorphs, break the rotors in the old way and place the beacon on the asteroid. This needs to be done quickly - oxygen supplies are rapidly falling, and we still need to have time to get back inside the ship.

Come back. The dust room is now in flames, but there are still safe places, and you can slip to the elevator (small, in the room to the right). True, you first need to return the power supply to the receiver from where you took it. It's not too difficult, the main thing is not to rush and throw the block forward with kinesis.

Take the elevator up to the control room, collect items and use the remote control to send the beacon with the stone into free flight. It is hoped that someone will receive the signal. However, even if this is so, we will not know the answer. The antenna is broken and needs to be repaired. Return to the transport station via the large elevator. You will be attacked by necromorphs - there are many of them, so do not lose caution.

Find and rescue

We have already been here, so you should easily navigate the terrain. Run to the bridge, not forgetting to visit the store first. On the bridge, an electronic security service will alert you of danger, close all entrances and exits, and leave you alone with the monsters. Various creatures will actively jump from the ceiling, among which there will be many demolition men. Considering that necromorphs can literally fall on their heads, do not stand still and constantly move.

After repelling the attack, you will remove the quarantine. Run into the room where the creature once jumped out, and take the power block from the box on the wall. Collect the items left over from the battle and go to the elevator. Go to the third tier. Kendra will contact you and tell you about the current situation. There are two guards in the next room, which is no joke at all. Prepare for this meeting with the utmost seriousness and care. There is a little trick - until you stick around the corner, the pods of the second monster cannot shoot at you.

From time to time you will meet friends, but unfortunately, these meetings are short and not very frequent. You will have to spend most of the time alone.

In the next corridor, one of the nastiest necromorphs is waiting for you - an octopus. Cut off his legs and kill the critters on his back. Don't forget to throw stasis to keep them from spreading. On the body of the octopus you will find a ruby ​​microcircuit. Scatter trash with kinesis near the entrance and go to the elevator. After reaching the next tier, pick up the power unit and ride the transport cart through the tunnel. This time no one will attack you. You will be taken to the communication room, from where you can go to the antenna itself.

Collect all the items in the control room and stomp to the antenna. Standing near the entrance, finish off the jumping necromorphs. Now take a close look at the device. Half of the batteries do not work, but the rest will be enough for us. Your task is to remove the non-working batteries and place the workers in such a way that a circle forms around the center. The room has zero gravity, so it's a matter of two minutes to assemble the devices. The antenna will then work and you will be able to receive the signal.

The warship has received an SOS signal and is heading for the Ishimura. True, he also picked up an escape pod, in which there was one of the creatures, which Hammond put there. It would be necessary to warn the military, otherwise the embarrassment may turn out. But the transmitting antenna doesn't work either. More precisely, it works, but something big prevents it from transmitting the signal. You need to quickly get rid of the monster and contact the ship.

Leave the room and ride the cart to the elevator. By the way, there will be a monster in it, so keep your weapon ready. Take the elevator down to the lower tier and run to the weapon control room. Collect the items, among them the power block, and sit in the shooter's chair. You need to destroy the monster that entangled the antenna. We'll have to shoot four tentacles. You need to shoot, as usual, at the yellow bags.

But the monster is not going to just give up. He will throw pieces of Ishimura's casing and all kinds of rubbish at you. The armor at the beginning of the battle is 100%. If it drops to zero, Isaac will die. Projectiles can be destroyed on approach, but it's not that easy. All four limbs will throw, try to keep track of all! It is better to focus on destroying the tentacles, trying to shoot down the debris as much as possible.

When you destroy the monster, Kendra will start transmitting a signal, but it will be too late. You will see how the captain of the ship is finished off by a necromorph. The warship will then crash into the Ishimura and Isaac will lose consciousness. When the character wakes up, Hammond will contact you and say that someone blocked his signal. Oddly enough, there is good news. It looks like a comrade has found a work shuttle.

To launch the ship, you need to sneak onto the wrecked warship and steal one important detail. The dark future is lit up again with the light of hope, and you need to hurry to the transport station. Along the way, you will encounter several necromorphs, including a malicious octopus. The last monster, it will be a gut, will generally be waiting at the transport.

This lanky monster is one of the nastiest necromorphs in the game. He does not move too fast, but he can hit with a tentacle from a decent distance or even cling to the character. It is not difficult to shoot him, but the death of the monster is only the beginning. After death, small octopuses separate from the back of the necromorph, which quickly crawl and painfully grab Isaac's head. They are even more dangerous than a swarm of bellies.

Try to shoot the octopus from afar. When monsters begin to crawl from the back, slow them down with stasis and bring down them until they crawl around the room. Or slow down the octopus, cut off its legs and, while it falls to the floor, burn the monsters from the flamethrower. However, if the flamethrower is without improvements, it will take a lot of fuel to kill octopuses.

Dead on arrival

Upon arrival, Hammond will contact you and say that the warship was not nearby for a reason. He purposefully flew to the Ishimura, but not for a rescue operation, but to destroy the ship. For some reason, this does not seem surprising. Go to the weapon room of the crashed ship. During the crash, there was a leak of radioactive elements, and until you clear the room of them, you will not be able to go further.

In zones with zero gravity, you can jump huge distances.

This is a zero gravity zone. Go down to the large hatch and destroy the six mechanisms on the sides. In order to save ammunition, you can kick. The hatch will open, and then all the air will disappear. Now carefully watch the amount of oxygen. Shoot the crawlers and use kinesis to collect and throw green spheres into the open hatch. From time to time you will be disturbed by necromorphs.

After clearing the room, jump to the exit and leave the zero gravity zone. Everything on the ship itself is broken and explodes. Therefore, you will constantly bump into zones with disturbed gravity. After going a little forward, you will see a monster running away. Do not shoot after him, he will not return anyway. In the cargo hold with kinesis, move the containers aside to go further. Soon you will meet new necromorphs - runners. Try to slow them down with stasis or chop off your legs to stop the enemy.

Turn left and disassemble the blockage in front of the door to go further. Keep moving forward, shooting monsters. Try to open the door that leads to the upper deck. You won't succeed, but necromorphs will appear from behind. Finish off the monsters, pick up the power supply and insert it into the receiver. This will activate the lift, which will take you to the upper deck.

On the way to the upper deck, you will be contacted by Dr. Kane, who will tell you that escape is not the best course of action in the current situation. You need to deliver the obelisk to the planet to end this madness, otherwise humanity is doomed. Go to the armory and shoot the octopus with its dorsal cronies. Deal with other monsters, collect items (there is one power block) and exit the warehouse.

However, if you are in no hurry, you can take part in a competition for accuracy and rate of fire. To do this, approach the shooting gallery on the left and use the remote control. You need to shoot off the red pieces and not touch the blue ones. You have no margin for error. For the first and second victories you will receive ammunition, for the third - a medium first-aid kit, for the fourth - a ruby ​​chip, and for the fifth - a power unit.

On the way to the medical compartment, you will meet a mortally wounded soldier. To get through the next room, you need to stasis freeze the laser installation and carefully slip past the deadly beams. Just before that, wait until all the curious monsters die, and collect the scattered objects. Run forward, shooting monsters, until you reach the barracks (just don't miss the power block).

Grunts need to be killed especially carefully so as not to let small monsters out of the belly.

Go inside and walk forward a little. The script will work, and you will be attacked by monsters. There will be too many of them. Move constantly and do not spare bullets, the battle is really difficult. Having defeated the necromorphs, start collecting trophies. In addition to items dropped from monsters, the hall is full of boxes, plus a lot of interesting things in the side niches. One thing is good - all unnecessary can be sold in the store, which is located here.

After reaching the engine room, take the elevator upstairs. You need to turn off the engines, and for that you have to turn off the power. The problem is that the room is periodically well fried. Take a close look at the incomprehensible barrel-shaped devices on the left and right. You can stand behind them without being hit by the flames. These devices will serve to protect us. Become an object and grab it with kinesis. Take a little to the side, then push forward.

Look carefully at the center of the room. See purple devices? They need to be destroyed with well-aimed shots. There are three of them on each side. Destroying everything, you turn off the flame and you can pick up the core. But this will be the last blow for the ship, which will rapidly fall to pieces. Run where the navigator points until you meet Hammond. This is your last meeting, because the creature will trample your comrade. Wait until the monster breaks the glass, brake it with stasis and shoot it in the back.

Take the diamond microcircuit from the corpse and quickly run out of the engine room. Remove the power supply near the elevator and place it in the old receiver to lock the door. Now run to the weapons room, where we got rid of the radioactive elements, fly to the exit and stomp to the transport station. On the way, monsters will attack you, but this is already a tradition that does not even surprise you.

Runners are the result of the transformation of the corpses of soldiers equipped with a stasis module into necromorphs. These monsters are strong, fast and twitchy. It is advisable to slow them down with stasis so that they do not run too much. But this is not always the case, because they move too quickly. Therefore, it is better to keep a powerful cannon ready to shoot down the runners on the way.

Last days

One core was not enough to launch the shuttle. We need to find three navigation maps. But in order to get into the room where they are stored, you need to get an access key. Go to the living room, where the crew members committed a mass suicide, and after finishing your household chores, exit through the door at the end of the hall. In the corridor through the glass you will see how the crazy doctor will finish off the surviving crew member and leave the room. Go to the dining room where the murder was committed and take the elevator to the lower tier.

Below you are waiting for two octopuses, which you need to quickly neutralize. Search the room, take the power block and the access key, and take the elevator back to the dining room. Dr. Kane will contact you and ask to meet with him when you find three navigation charts. Previously, he would not open the doors. Exit the dining room and go to the living quarters. Finish off the necromorphs and use the key you found to open the door leading to block A.

As soon as you step into the bedroom, Isaac will grab the tentacle and drag him down the hole. Shoot the orange knot to free yourself. Use Kinesis to open the magnetic lock, collect all the items and go to block B. In it, the climate control system failed, and the bedroom turned into a huge freezer. And without oxygen. Deal with monsters, replenish air supplies and go into the next room. Finish off the necromorphs again, pick up the navigation map and return to the hall where the door was opened with the key.

Demolitions will attack you in the hall. After coping with them, go to block C. Go to the basketball court. In the locker rooms, pick up the scheme for the level 5 armor. In the basketball court, this is a zero gravity zone - jump onto the court in front and grab a nautical map. In the room, sliders immediately materialize. Do not stand still, jump from platform to platform so that you are not bypassed from the rear.

If you want to get prizes, do not rush to leave the site. You can play space basketball. The rules are simple enough. With Kinesis, you grab a flying ball, jump onto a lighted area and throw the ball into the ring. You are limited by the total time and time for a specific throw. The prizes are as follows. The first and second rounds are ammunition, the third is the medium first aid kit, the fourth is credits, the fifth is the ruby ​​chip.

Leave the basketball court and stomp into block C. In the room where the mad doctor is broadcasting from the monitor screens about the need for self-sacrifice, finish off the monsters and go to the bedroom. You need to move the beds with kinesis so that you can go to the opposite side of the room. Just remember what and where to move back, because soon you will have to flee.

After picking up the last nautical chart, immediately turn around. The crazy doctor has set the hunter on you again. True, it is not clear whether it is old or new. Shoot off the limbs immediately and slow the monster with stasis. Quickly move the bunks back to leave the room and run to the exit. The door, as luck would have it, is closed, and until Kendra breaks the lock, we must keep the line.

Soon the obelisk, the monster and the whole colony will come to an end. A piece of the planet the size of a small continent will fall on them.

You will have to shoot back from ordinary necromorphs and a hunter. The latter must be constantly dismembered and slowed down with stasis so as not to get tangled underfoot. When Kendra opens the lock, run into the room where the crew members committed suicide. Do not forget to slow down with stasis and dismember the monsters you meet.

After reaching the room with the corpses, quickly find and destroy the two infestations. If you miss them, they will raise all the dead and have to deal with a horde of monsters. After clearing the room, sell the items in the store and go to Dr. Kane. He will open the door and tell you that all the monsters are telepathically controlled by the main monster that lives on the planet. The only way to stop him is to return the obelisk to the planet.

You need to get to the shuttle. You will go through the officers' cabins, which must be searched for valuable items. After reaching the shuttle, collect all the items, enter the ship and download the navigation maps. Nothing bad happened, which is a bit surprising. Go to the control room and use the remote control to activate the system.

You did it, but a hunter appeared from behind. Now everything is in order. Slow it down and run out of the control room. Other Necromorphs appeared on the dock deck. Deal with them, and then lure the hunter to the shuttle engines. Dismember, stasis slow, and run to the control room. Use the second remote to start the engines. The hunter will burn out and will not disturb you again.

All that remains is to release the clamps and watch Dr. Kane board the ship. Now you need to deliver the obelisk to the ship. Exit the control room, pick up the power unit from the hunter's body. Run to the transport station. You will again be bothered by monsters, but much more interesting is that a crazy doctor will contact Isaac, who will preach a sermon again and sacrifice himself to the contaminant. What an enthusiast ...

Alternative solution

Having reached the desired station, run into the cargo hold. There are few monsters. After reaching the remote control in the cargo hold, take the items from the boxes and the power block from the container on the wall and press the button to raise the obelisk from the bowels of the ship.

Pushing the cart at the end of the game is clearly overkill. It will have to drag as many as two levels.

You need to push the obelisk cart to the cargo hold. This is how it is done. Grab the cart with kinesis and drag it along the rails. As soon as you start moving, a crowd of necromorphs immediately materializes in the room. Also, tentacles will periodically rise from the bowels of the ship, which must be destroyed by shooting at the orange bags. After pushing the cart to the lift that will move the obelisk to the hangar, return to the transport station.

From there, run to the hangar, exterminating the monsters you meet. The loading system does not work, so you will have to load the obelisk manually. Run to the control room and turn off gravity. Now go down to the hangar, finish off the necromorphs and run to the obelisk. You need to take the cart to the hold of the ship. Twice, with the help of a computer, you will have to translate the switches on the rails in order to drag the cart. And of course, necromorphs will rush at you.

Return to the control room and turn on gravity. Now run to the ship to meet with Kane. True, it is not destiny to get to it. Kendra will shoot the doctor! She, it turns out, works for the government and must deliver the obelisk to Earth. Moreover, Kendra will tell you that the obelisk was created by humans. This is an unsuccessful copy from an alien artifact that was found on Earth. Here is the theory of alien intelligence has failed. Everything that happens is just the consequences of a failed experiment.

Nicole intervenes and calls Isaac to the mission control center. Run there - there is a guard on the wall, a lot of things in the lockers - and listen to what the main character's beloved has to say. Nicole figured out a way to stop Kendra, and soon the shuttle would begin to return to the Ishimura. True, the traitor was able to escape in an escape pod. But it doesn't matter anymore. Run to the ship. It's time to return the cursed artifact to the planet.

Jumpers have no legs and move exclusively by crawling. However, they do not suffer from this feature at all. You will see this for yourself when you see how fast they crawl along ceilings and walls. A jumper can just crawl up and hit with his tail, but he does not hesitate to attack in a jump. They especially love to do this trick in zero gravity zones.

The jumpers move fast enough, so try to slow them down with stasis. To hit a monster, you need to shoot off the hands and often the head. If you don’t want to waste stasis, wait until the monster crawls very close and shoot at your hands. When hit, the jumper will stop for a while, and you can finish him off.

Dead space

Exit the shuttle and pull the loader into the cargo area. Push the obelisk as far as you can along the rails and head into the room to the left. Open the room on the right and quickly finish off the infestations before they can break into the main room and turn a bunch of corpses into monsters. Then search all the boxes, pick up the power unit and grab the power supply. Reach the exit, throw it on the ground and go outside.

But Kendra was told that you shouldn't try to take the artifact off the planet ...

Shoot the octopus on the street, go back for the power supply and insert it into the receiver to supply power to the base gate. Go into any door that opens - the rooms are opposite each other and lead to one place - and push the obelisk inside. Use the remote to open the second gate inside the base.

Go to the warehouse. As soon as you take a step forward, a crowd of monsters will appear. Run around the room, shooting back at the monsters until they all die. You need to push the obelisk to the next checkpoint. This is done like this. Use the remote to lift the bridge. Until he let go, freeze the bridge with stasis and quickly carry the cart forward. Enemies will periodically pop out of ventilation as the obelisk progresses. One of the necromorphs will even drop a ruby ​​microcircuit.

Open and close the gate again to enter the next room. Go to the door on the right. There are two guards hanging there, which you need to quickly gut. The bridge controls do not work, so run through the door where the monsters were hanging. You will be caught in a long tube with zero gravity. Run upward, shooting off oncoming monsters, until you reach the very top. Use the remote to give power.

The fans started working along with the bridges, so the walk back will be more difficult. You need to not only shoot monsters, but also jump through the fans. To avoid being chopped to pieces, freeze them with stasis before jumping. A creature will attack you in the room. Follow the old scheme. First brake with stasis, then go around from behind and shoot in the back. You will no longer meet resistance, so you can safely push the obelisk to the excavation site. Just don't forget to collect all the items in the rooms.

Continue pulling the obelisk. On the left, three tentacles will crawl out of the abyss. Shoot the orange bags to destroy them. Then a crowd of monsters will attack you from the side of the base. Hold on to the defensive until you run out of necromorphs. Then collect trophies, push the obelisk to the very edge and activate the lift. Enter the room to the right and wait for the disinfection to finish. Perhaps we were in a hurry to leave, because Kendra appeared and, giving a speech, took the obelisk back to the shuttle.

When the monster grabs Isaac by the leg, continue shooting in the eyes. And do not hesitate, otherwise go straight into the mouth.

We urgently need to catch up with her. Run down the hallway. If you reach a room with a save point, a store, a workbench and a bunch of items, then the final battle awaits you behind the next door. Go outside and watch how the huge monster will finish off Kendra, and then, blocking the retreat with a tentacle, will take Isaac. However, the devil is not so terrible as he is painted, and you can cope with the main monster without problems.

The monster attacks with tentacles, just like the Leviathan. You need to run to the side to avoid blows and shoot at the yellow eyes around the mouth. When you destroy three eyes, the monster will grab Isaac by the leg and hang him upside down. This is not a cause for panic, so keep shooting from this position in the two remaining eyes. Depriving the monster of sight - even though they were eyes? - you will find yourself on the ground.

The monster will scream and start waving its tentacles more actively. However, not enough to stop us. Now you need to destroy the five yellow bags in the chest-mouth. They are harder to hit, so wait until the monster leans closer. I also advise you to stop throwing fuel cylinders. All the same, the tentacles will knock them out before you have time to make a throw.

By destroying the yellow bags, you will finish off the monster and clear the way to the shuttle. Hurry up to the ship and enjoy the not completely clear, but the final video.



Wait for the credits to pass and record. Now you can download it to start the game over, but with all the weapons and improvements that you made as you progress through Dead Space. You will also receive 50,000 credits, 10 power blocks and bonus armor, which can be purchased at any store for 99,000 credits.